OJ EPO SE 4/2015, p19 - PART II

BOARD OF APPEAL AND ENLARGED BOARD OF APPEAL CASE LAW 2014

I. PATENTABILITY

A. Exceptions to patentability

1. Breaches of "ordre public" or morality

(Case Law of the Boards of Appeal, 7th ed. 2013 ("CLB"), I.B.2)

In T 2221/10 the board had to examine whether the subject-matter of the appellant's (applicant's) request fell under the exceptions to patentability of Art. 53(a) EPC in conjunction with R. 28(c) EPC. Claims 1 and 2 of appellant's sole request referred to methods of maintaining human embryonic stem cells (HES cells) in culture in an undifferentiated state. Claim 5 referred to a cell culture comprising HES cells. HES cells are derived from the inner cell mass of human embryos at the blastocyst stage and can be proliferated in vitro in an undifferentiated state. They are capable of developing into any organ or tissue of the human body.

The appellant argued that methods using commercially or otherwise publicly available HES cell lines were not excluded from patentability because no de novo destruction of human embryos was necessary to perform them.

Regarding the appellant's argument that it would go too far if one were to take into account all the steps preceding an invention when assessing exclusion from patentability, the board referred to G 2/06, OJ 2009, 306, point 23 of the Reasons:

"In a case like the present one, where the teaching to obtain the embryonic human stem cells claimed is confined to the use (involving their destruction) of human embryos, the argument raised by the appellant, namely that the exclusion from patentability would go much too far if one would consider all the steps preceding an invention for the purposes of R. 28(c) EPC (formerly R. 23d(c) EPC) is not relevant."

The board interpreted this statement as meaning that for the purpose of R. 28(c) EPC, all steps preceding the claimed use of HES cells which are a necessary precondition for carrying out the claimed invention have to be considered. In this respect the Enlarged Board of Appeal had neither made a distinction between steps which have been carried out by the inventor or by any other person, nor between steps which took place in direct preparation of the experiments leading to an invention and steps having taken place at a point in time further remote from these experiments.

The board decided that inventions which make use of HES cells obtained by de novo destruction of human embryos or of publicly available HES cell lines which were initially derived by a process resulting in the destruction of the human embryos are excluded from patentability under the provisions of Art. 53(a) EPC in combination with R. 28(c) EPC. It noted that its decision was in line with decision C-34/10 of the ECJ.

2. Embryonic stem cells

(CLB, I.B.2.1)

In T 1441/13 claim 1 of the main request was directed to a method for obtaining polypeptide-secreting cells. The method required the use of a culture of primate pluripotent stem cells (pPS) which, according to the description of the application, included human embryonic stem (hES) cells. The board considered that at the relevant date of the patent in suit, the known and practised method for achieving cultures of hES cells, i.e. the starting material of the method of claim 1, included preceding steps that involved the destruction of human embryos. These destructive methods were not excluded from the scope of claim 1. Thus, in accordance with decision G 2/06 (OJ 2009, 306), the board decided that the main request was not allowable under Art. 53(a) EPC and R. 28(c) EPC.

3. Essentially biological processes

(CLB, I.B.3.3)

In T 1729/06 the invention was in the field of the production of watermelon fruit, in particular of seedless watermelons. Watermelon plants setting seedless fruits are produced by crossing a tetraploid inbred line as the female plant with a diploid inbred line as the male parent. The resulting triploid seeds can be germinated and grown into a triploid watermelon plant bearing triploid male and female flowers. Characteristically for triploid organisms, however, these triploid plants are female and male sterile. For triploid watermelon plants to develop fruits, pollination of the sterile female flowers is required. It is thus necessary to plant (diploid) watermelon polleniser plants in the watermelon production field to provide the necessary pollen to pollinate the triploid female flowers and thereby stimulate fruit development. The application was mainly concerned with the development and the use of particularly advantageous diploid enhanced polleniser plant lines and their use in the production of triploid watermelon fruits.

The claimed use and methods all concerned the step of the pollination of triploid watermelon plants by pollen of a particular diploid polleniser watermelon plant and resulted in the development of triploid seedless watermelon fruit on the triploid watermelon plants. They aimed therefore in essence at the production of triploid seedless watermelon fruit on existing triploid watermelon plants and not at the creation of any genetic make-up of any plant produced as the result of meiosis. The use and methods did not involve successful meiosis in the triploid plant flowers. Rather, they merely concerned the pollination of the sterile female flowers of the triploid watermelon plant with pollen of the diploid polliniser plant. They did not concern sexually crossing two whole genomes of plants (implying meiosis and fertilisation) and the subsequent selection of plants.

The board was therefore satisfied that the use and methods as subject-matter of the claims were not such methods which the Enlarged Board of Appeal in its decisions G 2/07 (OJ 2012, 130) and G 1/08 (OJ 2012, 206) had considered to fall under the exclusion of "essentially biological processes for the production of plants". The board noted that the Enlarged Board had not, in these decisions, given a comprehensive and exhaustive definition of the subject-matter to which the process exclusion in Art. 53(b) EPC applies in relation to plant inventions. Therefore, it needed to be established whether the claimed uses and methods were excluded from patentability by virtue of the process exclusion in Art. 53(b) EPC for other reasons.

According to the board, this was not the case. The board concluded that the legislator drafting Art. 53(b) EPC did not have the intention to exclude from patentability a whole class of inventions, i.e. horticultural or agricultural (agronomic) processes, under which the claimed use and methods undoubtedly fell. The EPC 1973 legislator (and the EPC 2000 legislator) only wished to exclude from patentability, in the context of the process aspect of the exclusion in relation to plant inventions, the – then conventional – processes applied by plant breeders in connection with new plant varieties for which a special property right was available under the UPOV Convention, and processes which were fundamentally of this type.

Thus, rather than being excluded from patentability by virtue of Art. 53(b) EPC, the claimed uses and methods constituted a "technical process" for which the EPC foresees patentability pursuant to R. 27(c) EPC.

In the consolidated cases G 2/12 and G 2/13 (OJ 2015, ***) the Enlarged Board was concerned with the question whether the exclusion of essentially biological processes for the production of plants in Art. 53(b) EPC had a negative effect on the allowability of product claims or product-by-process claims directed to plant or plant material (such as a fruit or plant parts) which are directly obtained and/or defined by an essentially biological process.

The Enlarged Board noted that Art. 31 and 32 of the Vienna Convention on the Law of Treaties are to be applied when interpreting the EPC. It applied the various methodical lines of interpretation which included grammatical, systematic and teleological interpretations as well as a consideration of the Biotech Directive. None of these lines of interpretation led the Enlarged Board to conclude that the term "essentially biological processes for the production of plants" extended beyond the processes to products defined or obtained by such processes. This result was confirmed when the preparatory work of the EPC was taken into account as a supplementary means of interpretation.

The Enlarged Board also examined whether considerations had arisen since the Convention was signed which might give reason to believe that a literal interpretation of the relevant provision would conflict with the legislator's aim. The Enlarged Board noted that the developments in the field of plant breeding techniques had not prompted the legislator to amend Art. 53(b) EPC. It stated that it could not see why the original intention of the legislator in drafting Art. 53(b) EPC was no longer justified, just because today there were new techniques available in this sector.

The Enlarged Board considered whether allowing patenting of a product claim or a product-by-process claim for a plant or plant material that is obtained by a means of an essentially biological process could be qualified as a circumvention of the process exclusion. It answered this in the negative, referring to the clear wording of Art. 53(b) EPC. The Enlarged Board warned that broadening the scope of the process exclusion to the extent that it included products obtained by essentially biological processes for the production of plants would introduce an inconsistency in the system of the EPC, as plants and plant material other than plant varieties were generally eligible for patent protection.

On the question whether it was of relevance that the protection conferred by the product claim encompassed the generation of the claimed product by means of an essentially biological process for the production of plants excluded as such under Art. 53(b) EPC, the Enlarged Board highlighted the distinction between the requirements for patentability and the extent of protection. Whether a product claim or a product-by-process claim was patentable was to be examined irrespective of the extent of protection that is conferred by it after grant.

As regards the ethical, social and economic aspects in the general debate, the Enlarged Board emphasised that considering these did not fall under the judicial decision-making powers of the Enlarged Board. It was not mandated to engage in legislative policy.

The Enlarged Board noted that it concurred with the District Court of The Hague in its judgment of 8 May 2013 and pointed to the differences in national legislation in relation to the patentability of plant products obtained by essentially biological processes.

The questions of law referred to the Enlarged Board of Appeal were answered as follows:

1. The exclusion of essentially biological processes for the production of plants in Art. 53(b) EPC does not have a negative effect on the allowability of a product claim directed to plants or plant material such as a fruit (G 2/12) or plant parts (G 2/13).

2. The fact that the process features of a product-by-process claim directed to plants or plant material other than a plant variety define an essentially biological process for the production of plants does not render the claim unallowable (G 2/13). The fact that the only method available at the filing date for generating the claimed subject-matter is an essentially biological process for the production of plants disclosed in the patent application does not render a claim directed to plants or plant material other than a plant variety unallowable (G 2/12, G 2/13).

3. In the circumstances, it is of no relevance that the protection conferred by the product claim encompasses the generation of the claimed product by means of an essentially biological process for the production of plants excluded as such under Art. 53(b) EPC (G 2/12, G 2/13).

4. Therapeutic methods

(CLB, I.B.4.4)

In T 1075/09 claim 1 as granted had been amended during the proceedings before the opposition division by including the feature relating to the administration of FSH. The appellant (opponent) raised an objection under Art. 53(c) EPC against claim 1.

The respondent (patent proprietor) submitted that the feature "and wherein folliculogenesis is induced by the administration of FSH" had been part of the definition of the patient group to be treated by the medicament of claim 1 and could not be considered to be a claimed therapeutic method involving a direct physical intervention of the human body. The claim was formulated in the Swiss-type format, accordingly no objection under Art. 53(c) EPC could possibly be raised.

The board noted that claim 1 concerned two different therapeutic methods, namely (i) the administration of FSH for the induction of folliculogenesis and (ii) the administration of LH for inducing paucifolliculogenesis or unifolliculogenesis in WHO Group II anovulatory women. Claim 1 was worded such that the purpose defined in the Swiss-type format was restricted to the administration of LH only while the administration of FSH was not covered by the Swiss-type format. Therefore claim 1 taught the direct administration of FSH to the patient for inducing folliculogenesis which feature constituted a method of treatment of the human body by therapy involving a direct physical intervention on the human body. The Enlarged Board had held in decision G 2/08 (OJ 2010, 456) that it was established case law "that any method claim containing even a single step pertaining by nature to a treatment by therapy is not allowable." The board concluded that claim 1 was excluded from patentability pursuant to Art. 53(c) EPC.

The board disagreed with the respondent's argument that the feature "and wherein folliculogenesis is induced by the administration of FSH" was part of the definition of the patient group to be treated by the medicament of claim 1. The board considered that the patient group was clearly indicated as being the WHO Group II anovulatory women and in the judgement of the board the wording of the feature "and wherein folliculogenesis is induced by the administration of FSH" left no doubt that the administration of FSH for the induction of folliculogenesis was carried out as an active step as part of the treatment of these WHO Group II anovulatory women.

B. Novelty

1. Information retrieved from the internet: proof of the date of availability

(CLB, I.C.2.5)

In T 286/10 the board did not share the conclusion reached in T 1134/06 that the prior publication of an internet disclosure had to be proved beyond all reasonable doubt. It found no legal basis for a standard of proof other than that applying to prior-art disclosures in general. Internet publications presented a special problem compared to conventional publications, given the potential for making changes that were not easily traceable; but that problem could not be solved by making an exception to the common standard of proof, which entailed establishing the existence of a prior publication by applying the balance of probabilities in the specific circumstances of the case (see T 2339/09, T 990/09 and T 750/94, OJ 1998, 32). To justify the need for a strict standard of proof for internet disclosures, the board in T 1134/06 had cited T 472/92 (OJ 1998, 161); yet T 472/92 too had confirmed the boards' consistent position that evidence was as a rule assessed on the balance of probabilities, and had made an exception only in the case of an alleged prior use where practically all the supporting evidence lay within the power and knowledge of the opponent.

In the case at issue the board held that the circumstances surrounding the internet disclosures cited by the appellant, i.e. A9 and A10, could not be likened to prior use which demanded a stricter standard of proof. Moreover, T 1134/06 itself had envisaged cases where a website belonging to a reputable or trusted publisher published online versions of paper publications, such that content and date could be taken at face value and the need for supporting evidence dispensed with. In other words, internet publications did not on principle call for a different standard of proof; any uncertainty linked to such disclosures had to be overcome in a way that ensured a sufficient degree of probability and established a presumption of availability that would convince the judge. There was no reason to raise the bar for probability and require proof beyond all reasonable doubt.

In the case in point the board held that www.jacksonville.com, operated by the "Times-Union" newspaper, was prima facie a known and reliable source of information, and it concluded that this was enough for it to presume that A9 had been published on 28 May 2000. Document A10 had apparently been archived by the Internet Archive www.archive.org on 9 July 2001. The board considered that the fact that a document had been archived by the Internet Archive on a certain date, naturally barring special circumstances that justified suspicion, usually sufficed by itself to warrant a presumption that the document had been normally accessible to the public on the day of its downloading and made available to the public via the Internet Archive itself shortly thereafter. The Internet Archive, a private non-profit archiving initiative, made past snapshots of the internet available to the general public. Since its creation in 1996 it had grown very popular and established a good reputation. The board therefore concluded that A9 and A10 had to be considered part of the prior art.

In T 1961/13 the examining division was of the view that document D2 was made available to the public before the priority date of the publication. In the contested decision the examining division did not rely solely on the date printed on the document. It considered that document D2 was made available to the public on or before 15 February 1999 through its publication on a website at the URL from which it had been retrieved. As evidence for this assertion, the examining division referred to document D7, which was a screenshot of search results returned by Google in response to a "quick search on Google cached pages" and which showed the URL to document D2 together with a date indication "15 Febr. 1999". According to the examining division, this showed that the URL to document D2 had been "snapshot" by Google on 15 February 1999.

The examining division's reasoning based on the screenshot shown in document D7 was refuted by the board. Document D7 showed the result, filtered by a data range, of a regular Google search for internet documents with part of the title of document D2 used as search criterion. The single result returned appeared to point to document D2 at the same URL from which it was retrieved. This did not mean that document D2 was cached (or "snapshot") by Google, but that Google at some point in time had found and indexed document D2 at this URL.

The board summarised that a date reported by Google is inherently unsuitable to serve as evidence of the publication date of a document. Such a date, extracted from the text of the document, adds nothing to what is already shown by the document itself. The board therefore considered that the evidence presented by the examining division was insufficient to conclude that document D2 was made publicly available before the priority date.

2. Content of prior art – taking drawings into account

(CLB, I.C.3.6)

In T 1488/10 the application related to an ultrasonic horn assembly stack component connector defined by the interface wherein the segment of the connection which is free of connection constitutes between 50% and 75% of an overall length of the connector. The examining division refused the patent application for lack of novelty. In particular, it had held that this feature of claim 1 had been disclosed in document D3.

Figure 4 illustrated the essential parts of the ultrasonic transducers, in particular, a stud connecting two masses (82, 84). Also shown in figure 4 were the longitudinal length L1 of the front mass 82 and the depth L2 of the slits 82a in the front mass. In view of the fact that the ratio of these lengths as measured from Figure 4 coincided essentially with the ratio of these lengths as described in the description, the examining division concluded that – at least in the longitudinal direction – the relative dimensions as shown in Figure 4 could be considered as conforming to the relative dimensions of the real device.

The board considered figure 4 to be a schematic drawing commonly used in patent applications. It stated that the length L1 of the front mass 82 as well as the depth L2 and the width t of the slits played a particular role in the ultrasonic transducers of document D3. Therefore, it could be expected that these lengths were represented in such a way in figure 4 that the ratio of their representations was in accordance with the ratio of the lengths mentioned in the description. As the ratio L2/L1 was described to have a value of between 1/3 and 1/2, the ratio of the representations could be expected to fall into this range as well. However, the dimensions of the other parts of the ultrasonic transducer shown in figure 4 had to be distinguished from L1, L2, and t. First of all, they were not referred to by any reference signs in the figure. Furthermore, they were not described as having any specific significance for the claimed invention. For example, no advantages were mentioned in the description concerning particular choices of the dimensions of these other parts.

Regarding the representation of the dimensions of these other parts of the device, factors like the clear visibility in the figure of these parts might well have induced the draughtsman to represent their dimensions in a manner that did not correspond to their actual dimensions. Therefore, in relation to these other parts of the ultrasonic transducer, the board was of the opinion that dimensions obtained merely by measuring the schematic drawing in Figure 4 did not form part of the disclosure of document D3 (see T 204/83, point 7 of the Reasons). Hence, feature (ii) of claim 1 of the main request had not been disclosed in document D3.

3. Ascertaining differences – functional features

(CLB, I.C.4.2.5)

According to the appealed decision, the sensor of D1 anticipated all the features of claim 1. The patent proprietor did not contest this finding except for the following feature F of claim 1: "wherein the sensor provides a measurement that correlates with the amount of analyte in a period of 10 seconds or less". In T 872/09 (see also under Chapter IV.E.5.2.2 "Request not submitted by proprietor in opposition proceedings") the board stated that the claimed sensor was defined by reference to characteristics of its response when used in a measurement set-up. Since none of the determining aspects of the measurement set-up was defined in claim 1, the technical features of the claimed sensor which were responsible for providing the measurement referred to in the claim remained obscure. Legal certainty required that a claimed subject-matter cannot be regarded as novel over the prior art on the basis of an ambiguous feature (see T 1049/99, point 4.4 of the Reasons). Hence, defining a functional feature of the claimed electrochemical sensor under undefined operating conditions was not appropriate to provide any distinction of the claimed subject-matter over the prior-art electrochemical sensors, all the more so if these conditions influenced the obtained result to a large extent, as in the case at hand. In view of the fact that feature F did not delimit the claimed sensor from the sensor of D1, it followed that the sensor of D1 anticipated the subject-matter of claim 1.

4. Second medical use

4.1 Reproducibility of the content of the disclosure

(CLB, I.C.3.11)

In T 1457/09 claim 4 was drafted as a second medical use claim. In the decision under appeal the opposition division held that the subject-matter of the corresponding claim was anticipated by the intermediate document (D1). The opposition division considered that both documents (D1) and (D1a, the priority document of D1) disclosed pharmaceutical compositions comprising the peptide RMFPNAPYL and their use as a cancer vaccine.

The board found that pursuant to established case law, a disclosure destroys novelty only if the teaching it contains is reproducible, in other words if it can be carried out by the person skilled in the art (see "Case Law of the Boards of Appeal", 7th ed. 2013, I.C.3.11, and in particular T 1437/07). For the requirement of reproducibility to be considered as fulfilled in relation to a medical use it is necessary that the disclosure in the prior-art document is such as to make it credible that the therapeutic effect on which the disclosed treatment relies can be achieved (see T 609/02). The board held that in the case at issue a prior art document was novelty-destroying only if it disclosed not only the product referred to in the claim – here RMFPNAPYL – for the claimed therapeutic application – here treatment of cancer – but also that the claimed product was indeed suitable for the claimed therapeutic application.

The board concluded that the experimental results disclosed in document (D1a) were not sufficient to make it credible that the RMFPNAPYL peptide was suitable for the treatment of cancer. Therefore document (D1) did not destroy the novelty of the subject-matter of claim 4.

4.2 Purpose-limited product claims and Swiss-type claims

(CLB, I.C.6.2.1)

In T 1780/12 the board examined in the context of the issue of double patenting whether the subject-matter of a claim directed to a new medical use of a known compound was the same, irrespective of whether the claim was in the "Swiss-type" format or in the format according to Art. 54(5) EPC.

The decision related to a divisional application with a main claim in the form of a purpose-limited product claim under Art. 54(5) EPC. Claim 1 of the parent application was granted in the "Swiss-type" format. The examining division held that the claims of the parent and the divisional applications were directed towards the same subject-matter "in the sense that both these claims concern the same invention claimed in a different format." It refused the application on the basis that double patenting was prohibited.

The board held that there was no double patenting. The decisive issue was whether the claims of the patent granted on the divisional and that granted on the parent had the same subject-matter. The category of a claim and its technical features constitute its subject-matter and determine the protection conferred (see G 2/88, OJ 1990, 93). The claims in question were of different categories: Swiss-type claims were purpose-limited process claims (use of X for the manufacture of a medicament for the treatment of Y) and claims formulated in accordance with Art. 54(5) EPC were purpose-limited product claims (X for use in the treatment of Y). As regards the technical features, the board concluded that both sets of claims defined the same compound and the same therapeutic use, but that the Swiss-type claims comprised in addition the feature of manufacturing a medicament whereas the claim in accordance with Art. 54(5) EPC did not. The claimed subject-matter was thus different. This outcome has been followed in decision T 879/12.

As regards the scope of protection, the board considered that a purpose-limited process claim, such as a Swiss-type claim, also conferred less protection than a purpose-limited product claim, such as a claim in accordance with Art. 54(5) EPC (for further details with regard to the scope of protection, see Chapter II.F.3.1).

In T 1570/09 the set of claims of the main request contained two independent claims, claim 1 in Swiss-type form and claim 4 as a purpose-limited product claim under Art. 54(5) EPC. The claims aimed to seek protection for one and the same medical use of one and the same active drug. The purpose-limited product claim 4 sought protection for the same medical indication of the same substance as the Swiss-type claim 1, and the notional novelty of claim 1 was not derived from the "medicament" itself. The board stated that there was no longer an objective reason for justifying the simultaneous presence of both claims in the set of claims to be proposed for grant. Allowing such a set of claims would cause the contradictory legal situation that the old provisions in Art. 54 EPC 1973 together with Art. 52(4) EPC 1973, and the new provisions in Art. 54 EPC together with Art. 53(c) EPC, would apply simultaneously to one and the same set of claims.

Art. 54(5) EPC applied to the case at issue and thus the purpose-limited product claim 4 of the main request was allowable in view of a new medical indication of a known substance. The Swiss-type form was conceived as an exception under the old law (EPC 1973). Therefore, there was no longer any legal reason for allowing Swiss-type claim 1 in the set of claims of the main request. Accordingly, the main request was not allowable.

C. Inventive step

1. Problem and solution approach

(CLB, I.D.2)

During the first-instance proceedings, the examining division found that D4 constituted the closest prior-art document, whereas the applicant considered D1 as being the closest prior-art document. In view of the amendments of the claimed subject-matter, filed by the applicant under cover of a letter dated 18 February 2014, the board in T 1437/09 considered that both D1 and D4 constituted a suitable starting point for the consideration of inventive step. In particular, both D1 and D4 belonged to the same technical field as the invention at issue, i.e. optical modulators, and disclosed the same type of devices. If there are several different prior-art documents, each of which might plausibly be taken as a starting point for the assessment of inventive step, it is established case law that inventive step be assessed relative to all these pieces of prior art before any decision confirming inventive step is taken (see T 967/97, point 3.2 of the Reasons; T 308/09, point 1.4.1 of the Reasons). The board came to the conclusion that the claimed optical modulator comprised an inventive step with respect to D4 as closest prior art and also with respect to D1 as closest prior art.

2. Closest prior art

(CLB, I.D.3)

On the issue of inventive step, the board in T 1760/11 chose document D2 as the closest prior art. The board rejected the petitioners' request to refer the following question (Art. 112(1)(a) EPC): in a case where there is more than one feasible starting point, is it admissible, contrary to T 21/08, to find an inventive step by applying the problem solution approach starting from only one of these starting points without considering the others? The board also dismissed the petitioners' objection under R. 106 EPC regarding their submission that the issue of inventive step should be addressed starting from document D1. In case R 5/13 (as well as R 9/13, R 10/13, R 11/13, R 12/13 and R 13/13 which were all directed against T 1760/11), the petitioners argued that they should have been allowed to discuss all the issues of inventive step of any stage of the problem and solution approach in respect of all possible starting points that they wished to rely on, despite the fact that the board had structured the discussion by first establishing which document or documents constituted the most promising starting point. The Enlarged Board held that the board had thus not only followed the sequence for the debate announced in its communication annexed to the summons to oral proceedings, but by doing so it had also systematically applied the standard method of the problem-solution approach.

The Enlarged Board in R 5/13 stated that the examination whether or not the subject-matter of a patent claim involved an inventive step according to the well-established problem and solution approach was a matter of substantive law. That was equally true for the determination of the closest prior art as the first step in the multi-stage method of the problem and solution approach, whether one document alone or a plurality of documents was taken as the starting point or most promising springboard aiming at the invention. The petition for review was not a means to review the correct application of substantive law (consistent case law).

The Enlarged Board observed further that of relevance to the present case was the principle of procedural economy, which required a board of appeal to focus on those points that are relevant for the decision. Consequently, the Enlarged Board did not follow the petitioners' argument that the board was obliged to let them continue to argue on the alleged lack of inventive step starting from document D1 despite the board's conclusion that that document was not to be taken as the closest prior art and the starting point for the further discussion of inventive step.

2.1 Similarity of the technical problem

(CLB, I.D.3.3)

The invention in T 25/13 related to a device for fastening an actuator and a housing, for use in a motor vehicle. D4, the only document cited by the opponent, which bore the burden of proof for lack of inventive step, described a fastening device for a tumble dryer.

The board observed that, to serve as the starting point for evaluating the inventive merits of an invention, a document had to relate to the same or a similar technical problem as the patent in suit, or at least to the same or a closely related technical field. D4, and more specifically the embodiment for tumble dryers shown in its drawings, came under domestic appliances, a field not even closely related to automotive technology, and for that simple reason could not be "the closest prior art". The skilled person attempting to solve a technical problem on a "closest prior art" basis might consult D4 as a source of secondary information, but only if the technical problem gave him reason to do so.

Opponents were essentially free to choose the starting point for assessment, but their choice then had implications for the technical knowledge of the relevant skilled person. The board found that in the case at issue there were two options: either to take the skilled person in the field of the invention – who would never have consulted D4 because it was from a totally different field – or to take D4 as the starting point, in which case the skilled person's field would be domestic appliances and it would not have been obvious to him to adapt the fastening device disclosed in D4 for use in a motor vehicle.

The opponent had gone for the second option, and the board held that its objections based on D4 had failed to establish that the subject-matter claimed lacked inventive step.

2.2 Speculative character of closest prior art

(CLB, I.D.3.4.6)

In T 1764/09 the board noted that the examining division in a first line of argument, had started from document D1 as the closest prior art, the selection of which as the closest prior art was questionable. D1 was no more than a speculative review of what might be potentially feasible in the future. No concrete realisation of a fully adapted lens for a contact or intraocular lens was described in D1. Therefore, for this reason alone, document D1 could not objectively be considered as a realistic starting point or the most promising springboard towards the claimed invention.

3. Technical problem

(CLB, I.D.4)

In T 632/10 the board stated that the application related to the question of how to provide long-term authenticity proof of electronic documents based on what is known as "electronic signatures" or, equivalently, "digital signatures". The board considered that an objective technical problem solved by the invention was to implement a digital signature system according to D1 suitable for long-term authenticity proof compliant with the requirements of Section 17 of the German Signature Law (SigV). During oral proceedings, the appellant (applicant) argued that the SigV might not be relevant for a European patent application such as the present one, for which states other than Germany could be designated, because compliance with German law was of no concern outside Germany. The board was unconvinced by this argument. Even if an invention happened to be obvious only for skilled persons of German nationality or residence, it would still lack an inventive step within the meaning of Art. 56 EPC 1973. The fact that the SigV was valid only within Germany thus had no bearing on its status as prior art or its relevance for the assessment of inventive step outside Germany.

3.1 Formulation of the technical problem

(CLB, I.D.4.3)

In T 2044/09 the board considered D1 as the most suitable starting point for the discussion of inventive step. The fermentor according to claim 1 differed from that of D1 in that it was linked to a recirculation conduit comprising sensors or analysers for measuring substrate components. According to the respondent (patentee), this difference over the closest prior art resulted in an improved process, in terms of better reproducibility, yield and process control. The board noted that, as admitted by the respondent, there was no data either in the patent or elsewhere on file to support the alleged effect. In view of the absence of any data confirming this alleged improvement, such an effect could not be taken into account in the formulation of the technical problem. The technical problem was thus formulated as the provision of an alternative U-loop fermentor/fermentation system. The board was convinced that the technical problem was indeed solved. It remained to be examined whether the skilled person would arrive at the solution in an obvious way.

The board noted that the mere fact that the claimed subject-matter was novel over the prior art, even when combining documents, was not sufficient to render it inventive. In fact, in the absence of a proven effect in comparison to the prior art, it was considered that this must be regarded as an arbitrary non-functional modification of the prior art. Even if there was no pointer or suggestion in the prior art towards the addition of a distinguishing feature, if said modification was not linked to a particular functionality then it could not per se constitute the basis for acknowledging an inventive step.

Document D6 was a declaration by a technical expert in the field. The board did not interpret this statement as evidence that there was a prejudice in the prior art, for which it would have to be shown that such a prejudice existed at the priority date; rather, D6 was taken as the opinion of one expert, issued almost ten years after the priority date. In any case, even if it were accepted that D6 demonstrated the existence of a teaching away from the claimed subject-matter, then the patent had not provided any evidence that a prejudice in general had been overcome. Lastly, even if the alleged improvement of the process had indeed been shown to be an effect of the modification of the closest prior art, if the skilled person expected some advantage of said features in a claim and obtained no more than this advantage, then the claimed feature combination was obvious (T 204/06). In the case at issue, it was not even confirmed that the expected advantage was achieved, let alone an unexpected effect. The board came to the conclusion that the claims of the main request did not fulfil the requirements of Art. 56 EPC.

4. Skilled person

4.1 Identification of the skilled person in the case of computer-implemented inventions

(CLB, I.D.8.1.4)

The board in T 407/11 held that the relevant skilled person in the context of providing computer-system users with operating assistance via a user interface (e.g. error messages or warnings) was an expert in software ergonomics concerned with the user-friendliness of human-machine interfaces rather than an expert in software programming or in computer technology in the strict sense. The objective problem to be solved by that skilled person was to prevent a situation whereby the user's action caused an electronic data-processing system to execute a called function differently from intended (or even to fail to execute it at all). In the board's view, however, the technical effect claimed in the application (simpler operation of an object-oriented user interface, facilitating initial use and subsequent familiarisation, especially for beginners or upgrading users, and so making the resulting method easier and more intuitive to learn) could not be considered a directly derivable consequence of the distinguishing features, because attributes such as "simpler operation" or "easy and intuitive familiarisation" were generally subjective, i.e. depended on the user's individual preferences and experience or intellectual capabilities, while the classification of users as "beginners", "upgraders" or "advanced" was generally based on a variety of criteria which were not clearly defined (points 2.1.3 to 2.1.6 of the Reasons).

4.2 Neighbouring field

(CLB, I.D.8.2)

According to the board in T 1910/11, whether the prior art and the claimed invention belonged to neighbouring technical fields within the meaning of T 176/84 was less a question of whether the relevant implementation parameters were identical than a question of the degree of similarity between the respective problems, boundary conditions and functional concepts. Applying this to the case in hand, it observed that automotive electronics and avionics were traditionally considered to be neighbouring technical fields because they involved similar problems (e.g. interference resistance, robustness and reliability), boundary conditions (e.g. mobility) and functional concepts (e.g. physical/logical separation of the communications systems for safety and maintenance data in the vehicle) (point 2.1.6 of the Reasons).

5. Assessment of inventive step

5.1 Treatment of technical and non-technical features

(CLB, I.D.9.1.4)

In T 2035/11 the application mainly related to navigation systems that could be tailored to a user's particular wishes. The focus of the application was on the route-planning functionality of a navigation system. The board held that the subject-matter of claim 1 lacked an inventive step within the meaning of Art. 52(1) and 56 EPC. It noted that mathematical algorithms may contribute to the technical character of an invention only in so far as they serve a technical purpose (see e.g. decision T 1784/06). The purpose of the algorithm was the mere display of an optimal path to the user for cognitive processing. The user could act on the information, but did not need to. As stated in decision T 1670/07, a technical effect may arise from either the provision of data about a technical process, regardless of the presence of the user or its subsequent use, or from the provision of data (including data that on its own is excluded, e.g. produced by means of an algorithm) that is applied directly in a technical process. In the case at issue, the data was produced by means of an algorithm and was not applied directly in a technical process, so neither possibility applied.

The board's reasoning did not rely on a particular suggestion in the prior art, but on the finding that the proposed algorithmic change had no technical motivation and that its implementation was, uncontestedly, trivial. In so far as it was the appellant's view that technicality was irrelevant for the assessment of inventive step, this view was contrary to the established case law. The board noted that the German Federal Supreme Court had come to a similar conclusion in respect of a navigation system offering the user the possibility to exclude road segments based on a user-selectable property such as the road segment being a toll road (see BGH, 18 December 2012, X ZR 3/12, GRUR 2013, 275 – Routenplanung).

5.2 Problem inventions

(CLB, I.D.9.10)

In T 764/12 the technical problem which the patent aimed to solve was, according to the respondent (patent proprietor), to provide chewing gums comprising degradable elastomeric polymers which did not degrade during storage and thus maintained their masticatory quality. As a solution to this problem the patent proposed the coated chewing gums of claim 1, including 0.1 to 75% of an outer coating. The board was satisfied that it had been credibly solved.

The technical contribution of the patent in suit resided in identifying a problem which was hitherto not recognised in the prior art, namely the need for protection during storage at ambient environmental conditions of a chewing gum base comprising environmentally degradable polymers. As held in T 2/83 (OJ 1984, 265), relating to so-called "problem inventions", the discovery of a hitherto unrecognised problem may in certain circumstances give rise to patentable subject-matter. According to that decision, this may be the case even in the situation where the solution claimed is retrospectively trivial and obvious.

Under these circumstances the arguments of the appellant (opponent) that the claimed invention lacked inventive step because coating was well known to reduce degradation over time were not relevant. The board acknowledged an inventive step not because the claimed solution was not obvious in view of the prior art but because the perception of the problem had to be considered as being the main contribution to the inventive merits of the solution claimed.

5.3 Examples of the denial of inventive step

(CLB, I.D.9.18)

In T 405/13 the invention was a meter with rapid response glucose sensor characterised in that the timing circuit was further adapted to cause the measurement of current to occur at a time 5 seconds or less after the detection of sample application. The objective technical problem to be solved was to implement a timing circuit having a concrete point in time at which the timing circuit caused the measurement of current to occur. Deciding whether the claimed solution to this problem was obvious or not boiled down to the question of whether or not the skilled person, while putting the meter of D10 into practice, would arrive at a value falling within the claimed range. A similar approach for assessing obviousness was taken in the decisions T 408/12 (point 7.3 of the Reasons) and T 315/97 (point 3.3.1 of the Reasons). In the board's view, no inventive step could be seen in including in the reasonably broad time range as required by the timing circuit of the meter of D10 the claimed value of 5 seconds after the detection of sample application, the less so since reducing measurement time had to be considered as an obvious desire of the user. The board decided that the subject-matter of claim 1 lacked an inventive step in view of the disclosure of D10 (biosensing meter).

The subject-matter of claim 1 was an at least tetravalent vaccine composition. The purpose of the claimed composition could be seen as providing a broad protection against HPVs, in particular those causing cervical cancer. The examining division considered document D8 to represent the closest prior art for the subject-matter of claim 1 but this was disputed by the appellant who considered document D10 as closest prior art. The board in T 1493/09 concurred with the examining division. As regards assessment of obviousness, the appellant (applicant) argued that the skilled person seeking to solve the problem formulated and therefore seeking to provide a vaccine effective in humans would not have contemplated directly applying the teaching of document D8 with respect to adjuvantation. In particular, it was argued that since document D8 only concerned animal studies, not suitable to predict protective efficacy in humans, the skilled person would have needed to seek further teaching with respect to this. However, in the board's opinion, the skilled person seeking to solve the problem formulated (provision of a broadly effective vaccine against HPV, especially providing broad protection against cervical cancer), would not be limited to considering only formulations for immediate use in human clinical trials. Additional animal testing such as described in document D8 could realistically be considered by the skilled person as a possible purpose for the claimed composition.

Taking into account the teaching of document D8 alone or in combination with document D10, the person skilled in the art would have arrived at the solution proposed by claim 1 of the main request without exercise of an inventive step. The requirements of Art. 56 EPC were therefore not fulfilled.

Claim 1 of auxiliary request 1 (directed to the second medical use of the vaccine composition) included the therapeutic effect ("prevention or treatment of a disorder related to HPV infection") as an explicit feature, while claim 1 of the main request did not. In fact, this effect (the same as the one as explicitly recited in claim 1 of the auxiliary request) was already taken into account by the board when evaluating the inventive step of the subject-matter of claim 1 of the main request. The board considered that the assessment of inventive step for the subject-matter of claim 1 of the main request according to the problem and solution approach applied to the subject-matter of claim 1 of the auxiliary request without any change in the reasoning.

D. Industrial application

(CLB, I.E.1)

In T 533/09 the examining division, in an obiter dictum to its decision refusing the application, had held that the subject-matter of claim 1 was not susceptible of industrial application, finding that an elementary pulse train as defined in claims 1 to 16 constituted a physical phenomenon which was not sufficiently tangible to be considered a product or process.

In stressing the need for an invention to be of a tangible nature, the examining division seemed more generally to have considered that the granting of a patent should be refused for inventions of an abstract nature. Art. 52(2) EPC's non-exhaustive list of non-patentable inventions did indeed appear to justify this approach. The board however did not agree with the examining division's conclusion, as it found that the claimed pulse train was of a tangible nature in that it was the result of modulating an electrical signal (derived from a capacitor discharge for defibrillation purposes) and its intensity was measurable at any time. Such a signal thus clearly came under the definition of "physical entity" within the meaning of G 2/88 (OJ 1990, 93), in its original version.

The board concluded that the nature of the subject-matter of claim 1 of the main request was no obstacle to its patentability; moreover, as there could be no doubt that such a signal could be produced and used for defibrillation purposes, that subject-matter clearly was susceptible of industrial application and thus met the conditions of Art. 57 EPC 1973.

The catchword of the decision may be rendered as follows: the wording chosen by the legislator for Art. 52 to 57 EPC on patentability does not in any way link the notion of patentable invention with any condition requiring such an invention to be of a tangible, viz. material, nature (see point 7.2 of the Reasons).

II. PATENT APPLICATION AND AMENDMENTS

A. Claims

1. Restrictions on generalising the technical information and examples adduced in the description

(CLB, II.A.5.2)

In T 2231/09 the patent in suit concerned a method of representing and analysing images. Claim 1 of the main request set out that "... at least one said descriptor element is derived using only a subset of pixels in said image." The board considered the expression "subset of pixels" to be problematic under Art. 84 EPC 1973 and stressed that, while a certain degree of generalisation may be permitted, features as claimed should make it possible to clearly identify features of embodiments that are covered by the terms of a claim. Moreover, the generalised subject-matter as claimed should make it possible to understand the technical problem to be solved.

When amending claim 1, the applicant had put forward a new interpretation according to which a "region" could mean the whole image, and a "subset" could correspond to all pixels of the region. The board considered this interpretation to be inconsistent with essential parts of the described embodiments, according to which a subset corresponded to only some of the pixels of a region. The subject-matter of claim 1 was thus unclear when interpreted in the light of the description.

The board also stated that the requirements of clarity and support by the description in Art. 84 EPC 1973 were designed to reflect the principle that the terms of a claim should be commensurate with the invention's technical contribution to the art. Taking into account the description, the board regarded the division of the image into regions and subsets as essential for achieving the technical effect underlying the invention. Therefore, the subject-matter of claim 1 was not supported by the description. The board concluded that claim 1 did not comply with Art. 84 EPC 1973.

B. Unity of invention

1. Unsearched subject-matter – subject-matter searched in divisional application

(CLB, II.B.6.2)

In T 2289/09 the application had been refused inter alia on the basis of R. 137(5) EPC. The applicant argued that because the subject-matter had been searched in a divisional application of the application-in-suit R. 137(5) EPC was met. This was consistent with EPO practice – R. 137(5) EPC made no reference to the timing of the search – and also in line with G 2/92, which stated that one could not pursue subject-matter in respect of which no search fees had been paid.

The board disagreed. The Enlarged Board had clearly stated in G 2/92 that when an applicant failed to pay the requested search fees, that subject-matter could not be pursued in the application for which the search was carried out. The Enlarged Board had stated furthermore that "the invention which is to be examined...must be an invention in respect of which a search fee has been paid prior to the drawing up of the European search report". This statement had significance beyond the reasons given in G 2/92.

The appellant had drawn an erroneous conclusion from the fact that claims in divisional applications may benefit from an earlier search performed in the parent application and erred in believing that this was also applicable in a different or "inverse" situation. When an applicant for a divisional application sought the benefit of an earlier search, he still had to pay the search fees (R. 36(3) EPC). These were then refunded, conditional on the fulfilment of certain criteria (decision of the President of the EPO and notice from the EPO, OJ SE 1/2010, 322 and 325), namely if the EPO benefited from the earlier search report. There was no provision that would permit the EPO to proceed with search activities, including the examination of the claims in question and their scope, when no search fees were paid for a given part of an application. There was also no provision that would entitle the examining division in the present situation to request the payment of further search fees for the purpose of examining the usefulness of the earlier search performed on the divisional application on which the applicant wished to rely.

C. Sufficiency of disclosure

1. Knowledge of skilled person relevant for assessing sufficiency of disclosure

1.1 The disclosure is aimed at the skilled person

(CLB, II.C.3.1)

If specific prior-art examples provide evidence of a technical prejudice, it is not enough, for the purposes of sufficiency of disclosure, simply to say that the prejudice has been overcome. The patent must provide guidance enabling the skilled person to identify the process features decisive in overcoming the prejudice. That person should not have to work out himself, from generally known information, the key points ensuring the success of the process over the entire breadth of the claim (T 419/12).

1.2 References may also enable a skilled person to carry out an invention

(CLB, II.C.3.2)

The enabling disclosure of the invention in T 521/10 relied on the content of patent applications incorporated by reference. Two US applications incorporated by reference did not fulfil the requirements set out in T 737/90 and Case Law of the Boards of Appeal, 7th ed. 2013, II.C.3.2, pp. 307 and 308. In order to be validly incorporated, each document must:

(i) be available to the Office on or before the date of filing of the application; and

(ii) be available to the public no later than on the date of publication of the application under Art. 93 EPC.

As neither of the two documents was made available to the public and the only publication originating from the two documents was a continuation-in-part application which was published on a date later than the publication date of the European application at issue, the two documents were not validly incorporated by reference.

2. Clarity and completeness of disclosure

2.1 Parameters

(CLB, II.C.4.5)

The board in T 2403/11 acknowledged that an ambiguity of a parameter in the claim was not enough in itself to deny sufficiency of disclosure. Whether such an ambiguity led to insufficiency of disclosure was to be decided on a case-by-case basis (see T 593/09).

The appellant had cited T 882/03, which also concerned viscosity. However, the variations resulting from the ambiguity of the intrinsic viscosity were only minor and the board in that case had therefore considered this ambiguity not to be such that sufficiency of disclosure had to be denied. In the case at issue, however, the variation of viscosity was by orders of magnitude, depending on which measurement frequency was used, and so was certainly not minor.

The appellant had also cited T 492/92 and argued that in the case underlying this decision a parameter was present in the claim that could be determined by two different methods and sufficiency of disclosure was acknowledged. However, in that case, the board had stated that if the skilled person was interested in the best possible accuracy of the results, he would know which method to choose. This was different from the case at issue where the method and the measurement parameters to be chosen were not known to the skilled person.

The invention was therefore held to be insufficiently disclosed.

In T 1697/12 the claims covered, through open-ended ranges, embodiments that could not be obtained with the process disclosed in the patent, but which might be obtainable with different methods still to be invented in the future. The board held that the monopoly claimed extended to subject-matter which, after reading the patent, would still not be at the disposal of the skilled person. The invention was therefore insufficiently disclosed.

3. Reproducibility

3.1 Reproducibility without undue burden

3.1.1 Chemical compounds

(CLB, II.C.5.6.9)

In T 544/12 the board confirmed that a definition of a group of compounds in a claim by both structural and functional features is generally acceptable under Art. 83 EPC as long as the skilled person is able to identify, without undue burden, those compounds out of the host of compounds defined by the structural feature(s) in the claim which also fulfil the claimed functional requirements (following T 435/91 and T 1063/06).

In the case at issue it was up to the skilled person to identify within the almost infinite host of alternatives covered by the structural definition of claim 1 those compounds that were phosphorescent. Claim 1 extended to classes (of iridium complexes) that were entirely different from the concept as argued by the proprietor. Granting a patent on this basis would therefore extend the technical contribution the patent made over the state of the art, contrary to the principle that the protection conferred by a patent should correspond to the technical contribution to the art made by the disclosure of the invention described therein (T 435/91).

The board was aware of the decision of the German Supreme Court (Bundesgerichtshof) X ZB 8/12 of 11 September 2013 stating that it was not objectionable as regards sufficiency of disclosure that a functional definition of a group of substances does not only cover known substances or those disclosed in the patent specification, but also those that will be made available in the future only, or those the provision of which may require an inventive step. However, the board did not share this view. As set out in T 435/91, in order for a functional definition of a group of substances in a claim to meet the requirements of Art. 83 EPC, the substances falling under this functional definition had all to be available to the skilled person. The invention therefore did not meet the requirements of Art. 83 EPC.

4. The requirement of sufficiency of disclosure in the biotechnology field

4.1 Clarity and completeness of disclosure

4.1.1 Repeatability

(CLB, II.C.6.1.3)

The claimed subject-matter in T 657/10 included an "elite event", i.e. a particular event resulting from a random method (for which the expectations always range from nil to high) and having at least one surprising, advantageous property. There was ample jurisprudence of the boards of appeal on "elite events". Although the specific random methods and resulting products with (normal) average properties might well be known in the prior art, the presence of a particular product with an unexpected advantageous property might justify the recognition of an inventive step. This jurisprudence on "elite events" was not limited to transgenic plants (see, inter alia, T 2239/08 and T 775/08) but had been applied in several other fields, such as those relating to monoclonal antibodies and to the isolation of specific virus strains or (yeast) microorganisms (see inter alia, T 645/02, T 1231/01 and T 737/96).

However, under Art. 83 EPC, the disclosure of the patent must enable a skilled person to carry out all claimed embodiments, including those concerned with "elite events". The disclosure has to enable a skilled person to obtain the particular product resulting from the "elite event" without the need to repeat the random method de novo, i.e. he must be able to obtain the particular product without having to rely on pure chance again. In the case before the board these requirements were not fulfilled.

4.2 Level of disclosure required for medical use

(CLB, II.C.6.2)

In T 1616/09 the board pointed out that, for the purposes of Art. 83 EPC, the level of disclosure in the application which is required for claims directed to pharmaceutical compositions or kits is not the same as that which is required for medical-use claims. For claims directed to pharmaceutical compositions or kits it is in principle sufficient that the application provides information which allows the skilled person to produce the composition or kit, and that there are no substantiated doubts that it could indeed be used in therapy. For second-medical-use claims, on the other hand, it is required not only that the composition itself is disclosed in an enabling way but also that its suitability for the claimed treatment is plausibly disclosed in the application.

In the case of a claim directed to a pharmaceutical composition comprising two classes of compounds which had both already been used in therapy in the prior art, there was a priori no reason to doubt that such a pharmaceutical composition could be produced; no specific functional effect had to be demonstrated.

In the case of second-medical-use claims, if the claimed therapeutic effect was already known to the skilled person at the priority date, it was not necessary to demonstrate it in the application.

5. The relationship between Article 83 and Article 84 EPC

5.1 Article 83 EPC and clarity of claims

(CLB, II.C.7.2)

In T 430/10 the patent concerned doped precipitated silica. The appellant argued that due to an unclear feature the claim could be interpreted as also covering further silica particles. It was therefore not clear for the skilled person whether he was working inside the scope of the claim. This argument was basically about distinguishing the claimed silica from others, especially the prior-art silica cited in the patent in suit. But that was more about interpreting the claims, i.e. their clarity under Art. 84 EPC, than about whether the invention could be carried out. And Art. 84 EPC is not a ground for opposition, and was therefore immaterial in the case at issue (see T 1062/98).

An unclear claim could indeed lead to insufficiency of disclosure. But a lack of clarity that prevented the skilled person from knowing whether he was working inside or outside the scope of the claim was not in itself enough to do so (see T 608/07, T 593/09).

If the lack-of-clarity objection was that a feature could be interpreted narrowly or broadly, the sufficiency requirements were met if the skilled person, applying the broad interpretation, was at least able to carry out all embodiments covered by the claims.

In the case at issue, the disclosure requirements under Art. 83 and 100(b) EPC were fulfilled.

D. Priority

1. First application

1.1 Identity of applicant

(CLB, II.D.3.1)

In T 1933/12 the application was filed by applicants A1 and A2. It claimed the priority of D0, filed by the same two applicants. Prior to D0, however, applicant A2 had filed application D1, whose priority was not claimed but which undisputedly disclosed a gear-box drive in accordance with claim 1 of the main request.

The board found that D1 – not D0 – was the first application within the meaning of Art. 87(1) EPC. Case T 788/05, cited by the patent proprietors, concerned the opposite case to that in hand, namely multiple applicants for the priority document but only one for the later European filing. For succession in title – as for first filings by multiple applicants but subsequent applications by only one or some of those applicants – it had to be known that the priority right held jointly by the multiple earlier applicants had been transferred to the sole applicant or the group of applicants by filing a subsequent application jointly with the latter (T 382/07; see also Benkard/Grabinski, EPÜ, 2nd edition, Art. 87, point 15; Bremi in: Singer/Stauder, EPÜ, 6th (2012) edition, Art. 87, points 41 and 56).

Art. 87(1) EPC did not exclude the possibility of the (sole) applicant for the first filing sharing his priority right with a third party with whom he jointly filed the subsequent application, but it did require that the applicant for the first filing, or his successor in title, be among the applicants for the subsequent, priority-claiming filing. In such a case, if there was only one additional applicant for the subsequent filing, there was no need to provide proof of the transfer of the priority right to him (see Bremi, loc. cit., Art. 87, point 56). In the case at issue, sole applicant A2 for the earlier application D1 was also one of the applicants for the application leading to the patent in suit, so D1 was the first application within the meaning of Art. 87(1) EPC.

The fact that D0 and D1 designated different inventors made no difference. Some of D0's inventors were also designated as inventors in D1, so it could not be assumed that the two applications derived from two different inventive acts. Most importantly, identity of inventor was not relevant for Art. 87 EPC, which, for the purpose of claiming a right of priority, referred to the applicant (T 5/05).

2. Partial priority

(CLB, II.D.4)

According to the catchword in decision T 571/10:

"In a case in which a single priority is claimed for a given application and a number of features of a claim of said application are generalisations of specific features disclosed in the priority document, a partial priority is to be acknowledged, as long as it is possible to conceptually identify, by a comparison of the claimed subject-matter with the disclosure of the priority document, a limited number of clearly defined alternative subject-matters, including among the alternatives the specific embodiments which are directly and unambiguously derivable from the priority document. In order for this condition to be met, it is not necessary that the clearly defined alternative subject-matters are spelt out as such in the application, nor that the word "or" actually occurs.

This condition extends to the case of multiple priorities. In that case, a comparison with the disclosure of each of the priority documents is necessary and for each of the clearly defined alternative subject-matters the earliest priority from which the alternative subject-matter is directly and unambiguously derivable is acknowledged."

The board based its reasoning on G 2/98, points 6.4 and 6.7 of the Reasons, and the additional remarks set out in T 1222/11, point 11 of the Reasons, both of which relied on preparatory document Doc. M 48/I, Memorandum C, drawn up by FICPI for the 1973 Munich Diplomatic Conference.

According to the facts, the application in suit and document D9 shared the same filing date and claimed priority from the same document. On that basis, D9 could belong to the state of the art under Art. 54(3) EPC only insofar as the priority of the application in suit was not validly claimed, while the priority of D9 was effective.

The board found that in the priority document two specific embodiments were disclosed for two of the features of the claim (a calcium salt as the active agent and a tribasic phosphate salt as the inorganic salt) with respect to the generic disclosures in claim 1. Thus, by comparing the claimed subject-matter with the disclosure of the priority document the board identified two clearly defined alternative subject-matters (a) and (b) covered by claim 1. The subject-matter of (a) comprised the calcium salt and tribasic phosphate salt and was fully disclosed in the priority document and enjoyed the claimed priority, while the subject-matter of (b) did not comprise those embodiments and was not directly and unambiguously derivable from the priority document and did not enjoy a priority right.

Turning to the priority of D9, the board here too identified two clearly defined alternative subject-matters (a) and (b), with the subject-matter of (a) being disclosed in the priority document and enjoying the claimed priority but the subject-matter of (b) not being directly and unambiguously derivable from the priority document and thus not enjoying a priority right.

The board held that for the subject-matter of claim 1 for which the priority was valid (alternative (a)), D9 did not belong to the state of the art under Art. 54(3) EPC, as it had no valid date prior to the priority date of the application under analysis. For this subject-matter, therefore, D9 was not relevant in the analysis of novelty. For the subject-matter of claim 1 for which the priority was not valid (alternative (b)), D9 was state of the art under Art. 54(3) EPC, but only for the subject-matter for which the priority of D9 was valid (alternative (a)). However, the subject-matter of alternative (a) of D9 was not novelty-destroying for the subject-matter of alternative (b) of claim 1, as the former had no overlap with the latter. As no lack of novelty arose, there was no need for a disclaimer with respect to D9.

E. Amendments

1. Article 123(2) EPC – added subject-matter

1.1 Implicit disclosure

(CLB, II.E.1.7.1)

T 598/12 concerned the issue of implicit disclosure in the (parent) application as filed. The board considered that what had to be judged was whether the notional skilled person working in the field would consider something as directly and unambiguously implicitly disclosed in the light of his common general knowledge. The assessment of what information was implicitly disclosed in an application could not go beyond the limits of what the skilled person would objectively understand to be a direct and unambiguous consequence of the explicit disclosure in the particular case. Going beyond that would allow the introduction of added matter, after the date of filing of the (parent) application, for which there was no unambiguous disclosure in the (parent) application as filed, contrary to the provisions of Art. 123(2) and 76(1) EPC. Moreover, when performing this assessment, the common general knowledge could not serve to enlarge or replace, in a subjective or artificial manner, the actual content of the specification.

The board also considered that the investigation of the actual disclosure in a patent application as filed pursuant to Art. 100(c) EPC could not turn into an investigation of obviousness or a search for obvious alternatives of the actual disclosure in the light of general prior-art documents. Such an investigation would be part of the assessment of inventive step under Art. 56 EPC, if the skilled person was looking to solve a certain technical problem without making use of his inventive skills, but it could not be taken as a valid approach for the investigation of implicit disclosure directly and unambiguously derivable from an application as filed within the meaning of Art. 123(2) and 76(1) EPC.

1.2 Intermediate generalisation – non-disclosed combinations

(CLB, II.E.1.2)

In T 1644/11 the examining division had held the subject-matter of claim 1 to be an intermediate generalisation which had no basis in the application as originally filed, relying in particular on T 284/94. The board could not endorse the conclusion of the examining division that the omission from claim 1 of certain features (the container is cylindrical, the container is a generally thin-walled hollow body, the cover has a circular surface, the vertical portion of the first flange is longer than the vertical portion of the second flange) extended beyond the content of the application as filed. By analogy to decision T 284/94, which stated that an amendment to a claim by introduction of a technical feature taken in isolation from the description of a specific embodiment had to provide a complete solution to a technical problem unambiguously recognisable from the application, the board found in the case at issue that the technical problem recognisable from the application consisted of the provision of eliminating the coupling – e.g. by welding or by buss bars – of the terminals to the battery, improving the lifetime of the battery, and configuring the terminals so that they could directly be coupled to terminals of adjacent batteries. Owing to this particular definition of the problem, the skilled person directly and unambiguously understood that the above-mentioned features did not contribute to the solution of the problem, and thus were not necessary for the definition of the invention. The features could be omitted from the subject-matter of claim 1 at issue without infringing Art. 123(2) EPC.

This conclusion was also in conformity with decision T 962/98 (point 2.5 of the Reasons) where it was held that an intermediate generalisation would be admissible if the skilled person could recognise without any doubt from the application as filed that the characteristics taken from a working example were not closely related to the other characteristics of the working example and applied directly and unambiguously to the more general context; it was also in conformity with T 273/10 (points 14.2 and 14.3 of the Reasons), where the board held that a claim amended by inclusion of a bundle of features extracted from a specific embodiment was allowable as long as the bundle of features proposed as an amendment comprised all the features essential for the performance of the invention, and that those features of the embodiment which did not contribute to solving the problem underlying the invention did not have to be part of the claimed subject-matter resulting from the amendment.

1.3 Disclaimer

(CLB, II.E.1.4)

In T 1441/13 claims 1 and 5 of the auxiliary request contained a disclaimer that excluded methods involving the destruction of human embryos, reading: "... wherein said pPS cells are not obtained by means of a process in which human embryos are destroyed". This disclaimer intended to overcome the objection raised against the main request which was found to be not allowable under Art. 53(a) EPC and R. 28(c) EPC (see also Chapter I.A.2 "Embryonic stem cell" above). The board considered that the criteria for the allowability of disclaimers were laid down in the decisions of the Enlarged Board of Appeal G 1/03 (OJ 2004, 413) and G 2/10 (OJ 2012, 376). According to decision G 2/10, the subject-matter remaining in the claim after the introduction of the disclaimer must be – explicitly or implicitly – directly and unambiguously disclosed to the skilled person, using common general knowledge, in the application as filed. In fact, this is "the overriding principle for any amendment to be allowable under Art. 123(2) EPC ... that applies equally to the subject-matter of a claim the scope of which is determined by a disclaimer", be it an undisclosed or a disclosed disclaimer.

In the case at issue, the subject-matter remaining in claims 1 and 5 of the auxiliary request after the introduction of the disclaimer, namely a method for obtaining polypeptide-secreting cells which included the culture of human embryonic stem (hES) cells derived only and exclusively from non-destructive methods, was not available at the priority date of the application (7 December 2001) and thus was not directly and unambiguously disclosed to a skilled person as required by decision G 2/10. Rather, the methods available to the skilled person at the relevant date all included, at some point in time, the destruction of a human embryo. In view thereof, the board did not consider it necessary to examine whether the disclaimer fulfilled the other criteria established in decision G 1/03, such as, for instance, whether the disclaimer was complete, i.e. whether it actually excluded all subject-matter not allowable under Art. 53(a) EPC in combination with R. 28(c) EPC. In conclusion, the board considered that the disclaimer was not allowable since the application as filed did not disclose the "remaining subject-matter" of the invention (a method which includes the culture of hES cells derived, only and exclusively, from non-destructive methods). Only with information available seven years after the claimed priority date would a skilled person have been in a position to put into practice this "remaining subject-matter".

1.4 Addition of limiting feature without technical contribution – legal fiction of compliance with Article 123(2) EPC – G 1/93

(CLB, II.E.1)

In T 1779/09 the board considered that the appellant had found itself exactly in the situation envisaged in decision G 1/93 (OJ 1994, 451). As emphasised in Headnote II of G 1/93, "a feature which has not been disclosed in the application as filed but which has been added to the application during examination and which, without providing a technical contribution to the subject-matter of the claimed invention, merely limits the protection conferred by the patent as granted by excluding protection for part of the subject-matter of the claimed invention as covered by the application as filed, is not to be considered as subject-matter which extends beyond the content of the application as filed within the meaning of Art. 123(2) EPC." These principles were confirmed in G 2/98 (OJ 2001, 413) and G 2/10 (OJ 2012, 376). The board in the case at issue concluded that a limiting feature which generally would not be allowable under Art. 123(2) EPC could, under certain conditions, nevertheless be maintained in the claim of an opposed patent in the particular situation addressed in decision G 1/93. It then complied with Art. 123(2) EPC by way of a legal fiction. In the case at issue, the term "only" was introduced during the examination proceedings and successfully objected to under Art. 100(c) EPC in proceedings before the opposition division by the former respondent. Since the board considered the term to be truly limiting, its deletion would extend the protection conferred and thereby infringe Art. 123(3) EPC. However, the board held that the exclusive limitation did not influence the solution of the technical problem as understood from the application as originally filed, and hence provided no technical contribution to the claimed invention (see also decision T 384/91, Headnote II). It merely excluded protection of part of the invention described in the application, thus not giving any unwarranted advantage to the applicant. Claim 1 of the appellant's sole request was therefore deemed to comply with Art. 123(2) EPC.

2. Article 123(3) EPC

2.1 Interpretation of scope of protection

(CLB, II.E.2.1)

In T 287/11 the question to be answered was whether the claims of the main request covered any compositions or methods which were not covered by the claims as granted. The board considered decision T 999/10, which had been cited by the appellant, where it was stated that in view of the sequential ("cascade") formulation of the claim, there was no doubt as to the "intention" of the patent proprietor that no other block copolymers other than the specific SIS-type might be present in the adhesive. That decision went on to state that even if the claim were to be interpreted as not excluding the presence of other block copolymers, the sequential formulation chosen by the patent proprietor meant that the condition limiting the amount of block copolymer defined in the broader manner as in granted claim 1 should also be fulfilled in the amended claim. The board in the case at issue held that the scope of protection should not be interpreted, however, in the light of the intention of the drafter of a claim, since this was a subjective criterion, but rather on the basis of the meaning generally ascribed by the person skilled in the art to the technical features defined in said claim. The board was thus not convinced by argumentation based on any alleged intention of a drafter of a claim. The board concluded that the main request did not satisfy the requirements of Art. 123(3) EPC.

3. Relationship between Article 123(2) EPC and Article 123(3) EPC, and interpretation under Article 69 EPC

(CLB, II.E.3)

In T 1736/09 the opponent argued before the board that, before deciding whether an amendment made in opposition complied with the requirements of Art. 123(3) EPC, it had to be established whether the text of the granted patent comprised subject-matter which extended beyond the content of the application as filed and/or the parent application as filed. The opponent added that any subject-matter comprised in the patent as granted but found to extend beyond the content of the application as filed or the parent application as filed must be excluded for the purposes of determining the extent of the protection conferred by the European patent according to Art. 69(1) EPC. There was thus a mutual interaction between the examination for added subject-matter and the determination of compliance with Art. 123(3) EPC. The board did not share this view. It referred to G 1/93 (OJ 1994, 451) where the Enlarged Board had held that paragraphs 2 and 3 of Art. 123 EPC were mutually independent of each other. The board considered that, by analogy, Art. 76(1) EPC and Art. 123(3) EPC must be regarded as mutually independent of each other. The board also disagreed with the opponent's view on the manner in which Art. 69(1) EPC was to be applied in this respect. The board summarised that, in determining whether amendments made in opposition proceedings complied with the requirements of Art. 123(3) EPC, the protection conferred by a European patent was determined by the claims of the granted patent, the description and drawings of the granted patent being used to interpret the claims in accordance with Art. 69(1) EPC and its Protocol. For the purpose of examination under Art. 123(3) EPC it was irrelevant whether the granted patent met the requirements of Art. 123(2) EPC or Art. 76(1) EPC.

F. Divisional applications

1. Added subject-matter

1.1 Omission of feature

(CLB, II.F.1.1)

In T 558/13 claim 1 according to the main request differed from claim 1 of the earlier application as filed, in that the following feature had been omitted: "... a plurality of power sources are independently provided for each of the plurality of groups." Instead, claim 1 of the main request specified that the read-out circuit comprised a plurality of sample and hold circuits. The board considered that the concept of the invention as consistently presented in the earlier application as filed required the provision of independent power sources. It accepted the appellant's argument that the shifted timing of the reset and sample and hold circuits provided the new technical advantage of "reducing a line noise caused by an external noise". Nevertheless, in the board's view, it was not sufficient to prove that the – now omitted – feature of providing a plurality of independent power sources was "not indispensable for achieving the effect of reducing line noise caused by external noise". Such reasoning did not take sufficient account of the information conveyed by the earlier application as a whole, which was primarily concerned with providing independent power sources and which presented the additional technical effect caused by the shifted timing only as a side issue. It might be true that a skilled reader, trying to improve the disclosed invention, could realise that shifted timing of the reset and sample hold signals might be sufficient to achieve a desired effect. However, what was required for complying with the provisions of Art. 76(1) EPC was a direct and unambiguous disclosure, in the (earlier) application as filed, of the subject-matter which was claimed in the divisional application. There was no room for speculation as to which features of the disclosed invention(s) might be omitted on further reflection, if there was no explicit or implicit disclosure of the generalised subject-matter remaining after the omission of these features. In other words, the disclosure of an independent technical effect that was achieved by some features of the subject-matter disclosed in the earlier application as filed did not suffice as proof of a direct and unambiguous disclosure of subject-matter comprising only the features which provided that independent technical effect.

1.2 Isolation of one species

(CLB, II.F.1.1)

In T 1919/11 the subject-matter of claim 1 of the main request related to a "cell culture of Taxus chinensis". The question was whether this feature was directly and unambiguously disclosed in the parent application as originally filed. The board observed that the subject-matter of claim 1 of the main request related to Taxus chinensis only. In contrast, the summarising text of the parent application described the subject-matter as relating to a plurality of Taxus species. In addition, any particular mention of Taxus chinensis was followed by an endorsement that Taxus chinensis might be the best choice in many respects, but that the important subject-matter of the application equally was the teaching to use any member of the list of Taxus species for producing taxanes. Some of the examples related to different Taxus species and some used Taxus chinensis as a model substance to show particular effects of the cultivation conditions. The board concluded that, on this basis, there was no direct and unambiguous disclosure that the subject-matter of the overall content of the application related to nothing else than a method using Taxus chinensis.

1.3 Arbitrary selection of upper and lower limits – jurisprudence on "ranges" not applicable

(CLB, II.F.1.1)

In T 1919/11 the subject-matter of claim 1 of the main request related to silver at a concentration of at least 1µM to less than 200µM. In the description, it was stated in two separate consecutive sentences that "When silver is incorporated in the medium, it will be added at a concentration of less than 900 µM, preferably less than 500 µM, and more preferably less than 200 µM" and "When silver is incorporated in the medium, it will be added at a concentration of at least 10 nM, preferably 100 nM, more preferably 1 µM, and typically at 10 µM". The board observed that the situation in the case at issue was not comparable with that in T 1107/06 and dealt with in the settled jurisprudence of the boards of appeal, where – starting from a general and a preferred range – "a combination of the preferred disclosed narrower range and one of the part-ranges lying within the disclosed overall range on either side of the narrower range" was considered to be originally disclosed. A general range, which means a lower limit which is unequivocally combined with an upper limit and a preferred disclosed narrower range – equally consisting of a lower limit which is unequivocally combined with an upper limit – were simply missing. Even a kind of parallel structure in indicating the upper and lower limits (less/at least, preferred or more preferred) implied no unequivocal correlation between a particular upper limit and a particular lower limit because there was no teaching that such an arrangement was intended. Therefore, one of the upper limits mentioned in the first sentence in the description of the parent application as originally filed (as cited above) and one of the lower limits mentioned in the second sentence were arbitrarily combined, which did not represent a direct and unambiguous disclosure.

1.4 Selection from lists

(CLB, II.F.1.1)

In T 1799/12 the appellant (opponent) referred to T 407/10 and the established jurisprudence on amendments arrived at by selecting features from different lists. The board considered that this jurisprudence established that such a selection in principle violated Art. 123(2) EPC. The premise was that the original application was not simply a reservoir from which the necessary features need only to be "tapped". However, the jurisprudence and T 407/10 correctly refer to further circumstances that need to be taken into account, such as pointers to that selection or combination in the description and in the examples, for instance the fact that the features in question have been mentioned in the description as "preferred". Further, the circumstance that the list consists of equivalent alternatives could also play a role (see in particular T 686/99). In the case at issue the patent as granted included the feature that the base wall had a "rectangular or square" shape. These circumstances were then also taken into account by the board in its assessment of whether this shape was directly and unambiguously derivable from the parent application. In that respect, for the skilled person, in the parent application was there not only no pointer available towards the choice of a square or rectangular base wall shape, but there was a clear pointer in the opposite direction, namely towards base wall shapes without any corners ("generally circular" or "oval"). By the same token, the five possible shapes mentioned in the parent application were not equivalent alternatives. The possible conditions in favour of allowing the currently claimed selection were therefore not fulfilled.

2. Final decision on subject-matter in the parent application – res judicata

(CLB, II.F.1.4.3)

In T 2084/11 the appellant (opponent) had argued that the invention's reproducibility was res judicata, a final decision (T 1832/06) having already been taken on the parent application. The board noted that the EPC did not mention res judicata, let alone regulate its applicability in the context of separate proceedings on parent and divisional applications or any resulting patents. EPO proceedings on a divisional were essentially independent of those on its parent (see T 1254/06). The board therefore doubted that a final decision taken in opposition appeal proceedings (here revocation of the patent) could have any "cross-procedural" res judicata effect at all on separate opposition (or opposition appeal) proceedings concerning a patent granted on a divisional. T 167/93 had already said that res judicata, whilst a generally recognised principle, was of extremely narrow scope and would only "involve something" where (amongst other criteria) "the issues of fact are the same". But it had said nothing about any "cross-procedural" application.

In the case in point, the board anyway took the view that the above criterion from T 167/93 was clearly not fulfilled, because the subject-matter (the "something" in the sense of T 167/93) requiring the division's or board's decision was not the patent granted for the parent application; rather, it was the divisional-derived patent and whether the requests submitted were legally sustainable in view of the objections raised, and – in contrast to T 51/08 – the requests (sets of claims) for decision in the two sets of proceedings were also not identical. Moreover, the underlying facts were different since further evidence had been submitted. The board's conclusion was that an appeal decision on the parent patent could have no res judicata effect in opposition (or opposition appeal) proceedings as regards the reproducibility, and thus the validity, of the divisional-derived patent in suit.

In T 1155/13 an objection of lack of sufficiency was raised by the appellant against the patent based on a divisional application. The board considered, however, that T 468/09 had dealt with an insufficiency objection in relation to the patent granted on the basis of the parent application. More specifically, although claim 1 of the patent granted on the basis of the parent application and claim 1 of the patent granted on the basis of the divisional application had a different wording, the board in T 468/09 had considered that the embodiment of figures 1 and 2 falling under the wording of claim 1 of the patent before the present board also fell under the wording of the claim at issue in that decision. Furthermore, documents D3 and D18 mentioned in connection with insufficiency of disclosure in the current appeal were the same as documents D7 and D29 used by the same parties in the former appeal proceedings and mentioned in the earlier decision T 468/09. Hence, since decision T 468/09 was res judicata and dealt with insufficiency of disclosure in relation to the same embodiments, and the facts were the same, the current board had no power to examine this objection again (T 51/08).

3. Double patenting

3.1 Swiss-type claim and claim under Article 54(5) EPC

(CLB, II.F.4)

In T 1780/12 the examining division had held that the application violated the prohibition of double patenting. It found that a claim directed towards the second or further therapeutic use of a substance or composition formatted under Art. 54(5) EPC and a Swiss-type claim directed towards the same therapeutic use of the same substance or composition were directed towards the same subject-matter "in the sense that both these claims concern the same invention claimed in a different format".

The board held that there was no double patenting. The decisive issue was whether the claims of the patent granted on the divisional and that granted on the parent had the same subject-matter. The category of a claim and its technical features constitute its subject-matter and determine the protection conferred (see G 2/88, OJ 1990, 93). The claims in question were of different categories: Swiss-type claims were purpose-limited process claims (use of X for the manufacture of a medicament for the treatment of Y) and claims formulated in accordance with Art. 54(5) EPC were purpose-limited product claims (X for use in the treatment of Y). As regards the technical features, the board concluded that both sets of claims defined the same compound and the same therapeutic use, but that the Swiss-type claims comprised in addition the feature of manufacturing a medicament whereas the claim in accordance with Art. 54(5) EPC did not. The claimed subject-matter was thus different.

The board also considered that the scope of protection sought by the invention claimed was noticeably different from the scope of protection conferred by the claims granted for the earlier application. It agreed with decision T 1391/07 where the board had considered that the lack of legitimate interest invoked by the Enlarged Board in decisions G 1/05 (OJ 2008, 271) and G 1/06 (OJ 2008, 307) could not be invoked in the case in which the scopes of protection conferred by the respective subject-matters overlap only partially with each other as there was no manifest objective reason to deny the legitimate interest of the applicant in obtaining a protection different from – although partially overlapping with – that of the parent patent already granted. As set out in decision G 2/88, the "determination of the 'extent of the protection conferred' by a patent under Art. 69(1) EPC was a determination of what was protected in terms of category plus technical features". The board held that it was generally accepted as a principle underlying the EPC that a claim to a particular physical activity (e.g. method, process, use) conferred less protection than a claim to the physical entity per se (see decision G 2/88). It followed that a purpose-limited process claim also conferred less protection than a purpose-limited product claim.

The board was thus satisfied that there was no manifest objective reason to deny the legitimate interest of the applicant in pursuing claims drafted in accordance with Art. 54(5) EPC and thereby obtaining protection different from – albeit partially overlapping with – that of the Swiss-type claims of the parent patent already granted. (See also T 879/12 agreeing with the conclusions in T 1780/12.)

For the avoidance of any misunderstandings, the board in T 1780/12 stressed that for the determination of whether or not the prohibition of double patenting applied in connection with divisional applications, the claimed subject-matter must be compared. This was different from the assessment under Art. 76(1) EPC, 87 and 123(2) EPC where the whole content of the (earlier) application as filed/previous application must be considered.

3.2 Product, use, and method claims – not the same subject-matter

(CLB, II.F.4)

In T 1765/13 the decision of refusal by the examining division was based solely on the ground that an unallowable "double patenting" existed. New claim 1 of the divisional application was directed to the use of a lithium secondary battery, comprising the specific cathode of the invention, as a high-power, large-capacity power source in an electric or hybrid vehicle, whereas the granted claims of the parent application related to products, namely a cathode active material and a lithium secondary battery. Thus the use claims of the divisional application related to a subject-matter different from the subject-matters of the granted claims of the parent application. The use claims were also distinct from the claims directed to a method for fabricating a lithium manganese-metal composite cathode, claimed in a co-pending application and found allowable in appeal case T 1766/13 (of 17 April 2014). The board held that, evidently, with respect to the case at issue, a product and its use were not "the same subject-matter" in the sense of G 1/05 (OJ 2008, 271) and G 1/06 (OJ 2008, 307). The same was true for the use of a product and a method of fabricating it. Therefore, the objection of double patenting could not be upheld against the claims as amended. The decision under appeal was set aside.

3.3 Overlap of scope of protection – internal priority

(CLB, II.F.4)

In T 2461/10 the examining division had refused to grant a patent because of the prohibition on double patenting. The board noted that the EPC contained only a provision relevant to this prohibition in the context of European and national applications and patents (see Art. 139(3) EPC). But double patenting issues could also arise with two European patent applications of the same filing or priority date. There were three possible scenarios: two applications filed by the same applicant on the same day; parent and divisional applications; and (European) priority applications and subsequent (European) applications claiming this priority.

Although the EPC did not explicitly regulate double patenting in the case of two European applications, the Enlarged Board had confirmed, in two identically worded obiter dicta, that it was not allowed (G 1/05, OJ 2008, 271 and G 1/06, OJ 2008, 307). In the light of these decisions, the reason for the prohibition was the lack of a "legitimate interest" (i.e. need for legal protection), which was in turn a general procedural requirement (see T 9/00, OJ 2002, 275) and one of the principles of procedural law generally recognised in the contracting states (Art. 125 EPC). T 1423/07 had held that double patenting was not proscribed for European applications claiming a European priority, because of the applicant's clear legitimate interest in the longer term of protection possibly available with the later filing. The appellant had argued accordingly in T 2461/10, which likewise concerned internal priority. But the board ruled that here the point could be left open; the examining division's decision had to be set aside anyway, because it had misinterpreted the scope of the prohibition on double patenting.

In the case at issue, the protection conferred by the priority patent and the application merely overlapped, and the board followed the case law holding that the prohibition should not be extended to include cases of overlap (see T 587/98, OJ 2000, 497; T 1391/07; in contrast to T 307/03, OJ 2009, 422). In this instance, therefore, the prohibition of double patenting was no reason not to grant a patent, and the board set the contested decision aside.

III. RULES COMMON TO ALL PROCEEDINGS BEFORE THE EPO

A. Legitimate expectations

 

(CLB, III.A.1)

In T 1448/09, in ruling on the appeal's admissibility, the board noted that the examining division's decision contained ambiguities which might have caused the appellant genuine confusion.

The principle of good faith that governed the relations between the departments of the EPO and the parties (see G 2/97, OJ 1999, 123) required adverse decisions not to contain any ambiguity that could affect their comprehensibility. That meant not only that their conclusions had to be clearly expressed, but also that the underlying reasoning had to be comprehensible without much need for interpretation. Otherwise appellants would not be in a position to comment on the reasons given.

In the case at issue, the board held that the appellant was not at fault in failing to address, in its statement of grounds of appeal, a reason which, in the circumstances, could genuinely have been misunderstood. It had certainly tried to address the reason, as (mis)interpreted. The board therefore ruled that the appeal was admissible.

B. Right to be heard

1. Non-attendance at oral proceedings

(CLB, III.B.2.3.3)

In the ex parte proceedings T 1367/09 (see also Chapter IV.E.2.2 "Patentability requirements under examination in ex parte proceedings") the board did not raise Art. 84 EPC in its communication pursuant to Art. 15(1) RPBA. When reconsidering the case in preparation for the oral proceedings, the board noted that Art. 84 EPC should also be addressed. The oral proceedings were held in the absence of the appellant. In its decision, the board stressed that a board's communication has a preliminary character and does not have to be exhaustive. In general, new grounds for refusal have to be discussed during the oral proceedings. However, if a duly summoned appellant does not attend the scheduled oral proceedings, it waives the opportunity to present its comments on new grounds which were not mentioned in the communication under Art. 15(1) RPBA but are decisive for the decision. In view of the principle of procedural economy, the board is not obliged to delay its decision. A board's decision dealing with new grounds, on which the appellant has not presented its comments, does not contravene the principle of the right to be heard (Art. 113(1) EPC) in such a case.

In T 1448/09 (see also Chapter III.A "Legitimate expectations"), refusal of the European patent application was based on the common general knowledge in the art as illustrated for example by document D3. That stance was first taken in the oral proceedings before the examining division, which the appellant did not attend. According to G 4/92 (OJ 1994, 149, points 8 to 10 of the Reasons), arguments could be presented at any time, even during oral proceedings in the absence of a party, but the same did not apply to new facts forming the basis for a decision. A reference to the common general knowledge could be presented as an argument, but the existence of that knowledge was a matter of fact. If its alleged existence was disputed, the facts relevant in that regard always had to be established. That however meant that the party against which this knowledge was cited had to have had the opportunity to dispute or accept it. In the case in point, the appellant was not aware of the examining division's invocation of the common general knowledge, or of the existence of document D3 cited in substantiation, until the adverse decision was announced. This violated the appellant's right to be heard on the relevance of document D3 and, by extension, on the existence of the invoked common general knowledge.

C. Oral proceedings

1. Continuation of oral proceedings on the next day

(CLB, III.C.4.1.8)

In T 1674/12, the opposition division had continued the oral proceedings on 9 September 2011, even though the summons was only for 8 September and two opponents had objected.

In T 2534/10 the board had ruled that continuation was possible only if a summons had been issued giving the minimum two months' notice under R. 115 EPC. So the opposition division had committed a procedural error, irrespective of the opponents' objections: to continue the proceedings on 9 September, it had to have the express consent of all parties. T 2534/10 had also ruled that remittal was necessary only if the error had affected the final decision, and had ordered the opposition division to resume the proceedings at the point at which it had committed it – a case could be remitted only to recommence procedural acts performed on the second day, not covered by the summons.

In T 1674/12, the minutes indicated that on 9 September the opposition division had opened the oral proceedings and announced that priority could not be validly claimed. At that point the patent proprietor had requested, and been granted, an adjournment. The oral proceedings had then been resumed on 18 April 2012, after the division had issued a summons in due form and time. If the minutes were correct, no procedural act affecting the parties had occurred on 9 September. The outcome of the division's deliberations could also have been communicated in writing. Nor had it adversely affected the opponents; their arguments had been accepted. The parties had been informed of this outcome both in the minutes they had been sent and by the opposition division's chairman, who had reiterated it when reopening the oral proceedings on 18 April 2012. Even if the division had not continued the proceedings on 9 September 2011, it would have communicated the outcome of its deliberations in the usual way, in the minutes or the summons to the new oral proceedings. In other words, the proceedings would have taken essentially the same course, even if the division had adjourned the oral proceedings on the evening of 8 September 2011 rather than continuing them the following day.

2. Location of oral proceedings

(CLB, III.C.4.5)

In T 1142/12 the examining division refused the request of the applicant to hold oral proceedings in Munich rather than in The Hague on the grounds that according to Art. 18(2) EPC oral proceedings were to be held before the examining division itself and consequently had to be held at the location of the examining division to which the case was assigned, i.e. The Hague in this case. The board considered that the crucial issue was to determine who had the power to decide on the location of oral proceedings, namely, the examining division or EPO management. As stated in T 1012/03, the practical aspects of the organisation of oral proceedings were matters of EPO management, which came under the power of the President of the EPO as provided by Art. 10(2) EPC. The board held that, accordingly, the examining divisions were clearly not allowed to take a decision, whatever this would be, on this matter. The mere decisional power of the first-instance bodies as well as the boards of appeal on this issue extended to the point at which the decision to hold oral proceedings in a specific case was taken. By contrast, the place, the room and even the date were of organisational nature. When not acceding to a request for holding oral proceedings in Munich instead of The Hague, the examining division did not take a decision but only expressed the way the EPO was managed. Consequently, that issue was not subject to appeal, nor could the board refer a question on the venue of oral proceedings to the Enlarged Board of Appeal.

3. Handwritten amendments during oral proceedings before the board

(CLB, III.C.4)

In T 1635/10 the board took note of the notice from the EPO of 8 November 2013 (OJ 2013, 603) regarding the practice on handwritten amendments to patent documents. From the accompanying information on the EPO website it was clear that this applied as from 1 January 2014 to first-instance proceedings. This practice is that handwritten amendments can be filed during oral proceedings, but for finalising the decision they should be filed there and then in typed printed form of the patent documents, or that a time limit should be set to file the amendments in such form. Both procedures require a check by the competent department that nothing more has been amended than what was allowed and giving the other party/parties the possibility to check and comment, before a decision can be issued. In the case at issue, the documents as proposed for the maintenance of the patent according to auxiliary request 3 contained handwritten amendments, carried out at the oral proceedings before the board. For the board to do what was required by the above notice would, in the present case, be detrimental to the principles of due process and procedural economy. The board therefore found that the documents as presented at the oral proceedings, for which the above checks could easily be performed and on which the appellant had had the opportunity to comment, were sufficient to base its (final) decision on. Since the department of first instance in any case would have to perform further formal steps before it actually maintained the patent in accordance with the board's order, it could apply its new practice to the present documents. (See also T 37/12.)

D. Re-establishment of rights

1. Error in making payment

(CLB, III.E.4.4)

In T 1355/09 the appeal had been received in due time, but not the fee. The board granted the appellant's request for re-establishment in respect of the time limit for filing an appeal. Under the boards' established case law, an isolated error in an otherwise well-functioning system was excusable. In the case at issue, the error had occurred not in monitoring the time limit but in making the payment. It made no difference that the reason for non-payment could not be established. Rather, even if the circumstances remained unexplained, the error could be regarded as an isolated one because the system had worked for many years (following T 529/09 and T 580/06). Nor did a control mechanism have to be in place, first because the firm involved was very small. Secondly, it had not overlooked a time limit; the mistake had been made during payment, i.e. whilst performing the act needed to observe the time limit. In these circumstances, no control mechanism was required, the likelihood of error being comparatively low (see also T 836/09, which found there was also no obligation to monitor outgoing mail). Insisting on one would in practice reduce the time limit: to be effective, any checks would have to be made after payment, but also within the time limit.

2. Isolated mistake by representative not excusable

(CLB, III.E.4.4)

In R 18/13 the petitioner requested re-establishment in respect of the period under Art. 112a(4), second sentence, EPC. It argued that this was a matter of isolated error within an otherwise well-functioning system for monitoring time limits. Two "isolated" mistakes had been made, one by the professional representative, the other by his assistant. The Enlarged Board, sitting in its three-member composition pursuant to R. 109(2)(a) EPC, ruled that a representative had a greater duty of care than his assistants. It cited J 5/80 (OJ 1981, 343), in which the Legal Board had said "one could not expect of an assistant entrusted with routine tasks the same rigorous care as is demanded of an applicant or his professional representative". The Enlarged Board found that an "isolated mistake", such as assistants might make, was less excusable in a representative. If he was given a file to deal with, he could not simply assume that his assistants had reliably done all their work on it so far; he had to make sure, through appropriate control mechanisms before the file reached him, that time limits arising were complied with, or – at the latest when he was given the file to deal with – check for any time limits himself. The request for re-establishment was refused.

3. Representative's secondary responsibility

(CLB, III.E.4.5)

In J 5/13 the board concluded that the representative had exercised all due care to meet the missed time limit. The board considered that the applicant had to accept the actions of his representative, including the actions of the attorney's assistants and employees, on his behalf (J 5/80, OJ 1981, 343). However, the extent of the duties of the representative depended on the agreement between the representative and his client. An appointed representative whose authorisation was silent concerning the payment of the renewal fees and who had not received any funds for this purpose was not expected to pay the fee by advancing money on behalf of the applicant out of his own pocket (J 16/93; J 19/04; J 1/07). Instead, he retained only a "secondary responsibility" (cf. J 1/07) to advise the applicant properly either if the applicant addressed him or if he became aware of any problem that might affect the applicant's position in respect of the patent application. Thus the representative's responsibility, above all, consists in finding out what his client really intends to do with respect to payment of the renewal fees (cf. J 16/93). Since the representative remains responsible in the procedure before the EPO and thus has to take the necessary steps to ensure payment, this includes a reliable monitoring system and sufficient reminders to the applicant (J 11/06, J 1/07). Accordingly, the scope of duties of a representative who retains only such a "secondary responsibility" to inform and advise his client with respect to the due date for renewal fees cannot be the same as it would be if he were responsible for the payment itself. In the case in question, the representative had fulfilled his responsibility by sending repeated letters to the responsible person within the appellant's company.

4. Request for re-establishment of rights redundant – statement of grounds minimally late-filed

(CLB, III.E.5)

In T 2317/13 the time for filing the statement of grounds of appeal expired on 12 November 2013. The appellant filed the statement by fax. The fax transmission began at 23.58 on 12 November 2013 and concluded at 00.16 on 13 November 2013. The appellant requested re-establishment of its rights into the time for filing its statement of grounds of appeal. The board held that the re-establishment request was redundant and there was no need to consider it on its merits. According to the board, the discretion given to the board by Art. 13(1) RPBA to admit and consider late-filed submissions which amend a party's previously filed written case extended to the late admission and consideration of the written case itself (see T 1198/03). The board observed that the delay in filing the statement of grounds of appeal was only of minutes and the lateness of the filing was truly minimal. Further, the first four pages of the statement of grounds were in fact received before the time limit expired and those pages were on any view admissible. It followed that the late-filed remainder should be admitted in order to make sense of the pages filed in time. Not to admit and consider the grounds of appeal purely because of that minimal delay would, in the circumstances of the case, have been an incorrect exercise of discretion. The board also decided to reimburse the fee for re-establishment. It compared the case with the one in T 152/82 (OJ 1984, 301) in which the fee had been reimbursed after it became apparent that the re-establishment request would not have become effective.

5. Reimbursement of fee for re-establishment

(CLB, III.E.8)

In T 2017/12 (OJ 2014, A76) the appellant had missed the time limits for filing the appeal and for filing the statement of grounds of appeal. It had paid the fee for re-establishment of rights twice, once for each missed time limit. It requested the refund of one of the fees due to the interrelation between the two time limits and since they both related to one and the same loss of rights. The board refused the request. There were no explicit provisions in the EPC dealing with the case in which several time limits had been missed. This was an indication that each time limit had to be considered separately and that, in the absence of any hint to the contrary, for the number of fees to be paid the number of missed time limits was decisive. In line with decision J 26/95, the board considered that the corresponding time limits expired independently of one another, notwithstanding the fact that they were triggered by the same event. In addition, the failure to meet either of these time limits individually results in a loss of the right to appeal. Failure to comply with either one of the two time limits would cause the appeal to be rejected as inadmissible, provided that the appeal fee was paid. Consequently, two fees for re-establishment were indeed due and hence a refund of one of those fees was not possible.

E. Law of evidence

1. Witness testimonies and expert opinions

(CLB, III.G.2.2)

In T 86/12, the appellant (proprietor) had first requested only a month before the oral proceedings that an expert be heard to help clarify how the skilled person would understand a particular term ("residual expansion").

The board conceded that expert opinions were listed in Art. 117(1)(e) EPC as a possible means of taking evidence even in opposition and appeal proceedings. However, it was generally a matter for the technical boards to assess the patent-law implications of technical facts they were able to understand, which was why R. 117 EPC said the Office had to obtain such an opinion only where it considered it necessary. There was no need to do so here, as the board's own technically qualified members had enough expertise in the relevant field and, by submitting another patent specification and a scientific study, the appellant had already made a case as to the term's usage. The technically qualified board therefore had an adequate basis to interpret it.

The request also had to be dismissed under Art. 13(3) RPBA because it had been filed at such a late stage that the expert could not have been summoned without adjourning the oral proceedings.

The appellant in T 1028/11 alleged that the opposition division had infringed its right to be heard, and so committed a procedural violation, by refusing to hear the witness it had offered. The board disagreed: a witness's function was, in essence, simply to corroborate the alleged facts on which they were heard and not to fill in gaps in those facts. Parties therefore had to specify what legally relevant facts the witness's testimony was intended to prove.

Based on the evidence available, the opposition division had taken the view that the public prior use was not prejudicial to patentability. The board, reiterating the witness function outlined above and observing that none of that evidence had included the essential feature at issue, found that there would have been little point in hearing the witness at that stage, and that this situation had not changed by the time of the oral proceedings before the opposition division. In those proceedings, the appellant (then opponent) had again requested that the witness be heard, setting out in detail how the missing feature followed from the witness's sworn statement produced with the notice of opposition. Given the witness's absence, however, the opposition division could not have granted the request without adjourning the proceedings.

Thus, at that late stage of the proceedings, the opposition division had been entitled to dismiss the request and had not thereby committed any procedural violation. On the appellant's request that the witness now be heard on appeal, the board explained again that a witness's function was not to introduce new assertions of technical facts with a view to making a convincing case but rather to prove an already convincing case. It was therefore not for the board to question the witness in order to fill in gaps in the insufficiently substantiated allegation of prior use. For that allegation to be adequately substantiated, the relevant facts had to follow already from the reasons for opposition and not only in the light of the witness's testimony. In the board's view, the appellant had not presented a "convincing case" which could have been corroborated by the witness. Since the evidence already adduced, which said nothing about the missing feature, did not make for a convincing case, the requirements for hearing the witness were not met and the request had to be dismissed.

T 8/13 addressed several procedural aspects. On the request formulated by the appellant (opponent) to hear an independent expert of a university, the board considered that the questions on which it had to decide did not require any further technical expertise to be gathered from outside the Office. Notwithstanding the fact that the board finally reached a different conclusion on the alleged obviousness of the claimed subject-matter than the appellant, clearly the board and the appellant had anyway drawn the same conclusions on the technical effects that a re-location of the heating element would have. The presence of two technically qualified members on the board in addition to the legal member, and the specific division of technical subject-matter between the various boards of appeal, was already provided for such that the parties' arguments, insofar as these were clearly formulated, could be fully understood from both a technical and legal standpoint. Also, all the board members were not only impartial, but also experienced in judging what knowledge was attributable to the notional skilled person, as opposed to an opinion provided by a technical expert which might be based on that person's subjective analysis. The appellant did not provide any further argument as to why hearing an independent technical expert would have been required in order to decide on this case. There were thus no good reasons why the board should have exercised its discretionary power under R. 117 EPC as requested by the appellant.

In case T 30/12 concerning alleged prior use, as evidence adduced for proving the invoked public prior use in opposition proceedings, the appellant (opponent) relied upon the drawing A9 and the testimony of Mr H, made before the opposition division. The witness explicitly admitted that he had no knowledge of when or to whom the drawing A9 was handed over (see the detailed minutes of the witness hearing, Mr H). In fact, Mr H did not testify that he experienced first-hand the actual handing over of A9 but only stated what he thought had happened. The board could not find that the opposition division was mistaken when it came to the conclusion that the testimony of Mr H was not sufficient proof of the fact that document A9 was made available to the public before the priority date of the patent in suit. The board had no reason to set aside the evaluation of evidence by the opposition division and to come to a different conclusion.

As regards the request for a renewed hearing of the witness Mr H before the board, the board noted that the appellant did not request that the witness be heard again to supplement his testimony by corroborating facts but only to clarify the statements made before the opposition division. However, the statements made by the witness as to whether document A9 was handed over to a member of the public were clear and unambiguous. Under these circumstances, the board judged that the request was not justified. The appellant also submitted that the witness, if heard, would have had the opportunity of explaining how his previous statements should be interpreted. However, the relevant statements did not leave room for interpretation. Moreover, the professional representatives of both parties were given the opportunity to question the witness in oral proceedings before the opposition division, whereby the opponent's representative could have asked the witness to clarify relevant statements, where the need arose. The request to rehear Mr H was therefore rejected by the board.

2. Evaluation of evidence

(CLB, III.G.4)

In T 2357/12 concerning the transfer of opponent status and the concept of "universal succession", the board commented on the evaluation of evidence. The proprietor questioned the probative force of private documents, as submitted in this case, rather than public registration documents, as evidence of the transfer of opponent status in inter partes proceedings. The board stated that whereas public registers often enjoyed public trust regarding the facts registered and other public documents might be more conclusive on formal questions, the identity of the issuer and the date and place of creation of a private document could be more easily contested. However, neither kind of document provided irrefutable evidence of the correctness of a document's content. Furthermore, the accuracy of public registers was dependent on the accuracy of the information received by the authorities, handed in mostly in the form of private documents.

In most cases, private documents seemed to be the most direct pieces of evidence. Citing examples of the different types of documents accepted in individual cases, the board pointed out that the boards had always accepted public and/or private documents as evidence of the transfer of rights, whether by way of universal succession or a single transfer of assets. The level of proof required was credibility of the facts for which evidence was given, in the light of all circumstances.

In G 1/12 (OJ 2014, A114) the board stated that proceedings before the EPO were conducted in accordance with the principle of free evaluation of evidence. The principle of the free evaluation would be contradicted by laying down firm rules of evidence defining the extent to which certain types of evidence were, or were not, convincing.

3. Burden of proof

(CLB, III.G.5.1.2)

In J 3/14 the board stated that in a situation where the representative submits that he has not received a communication, the EPO bore the burden of proof. In the case at issue the receiving section's communication, in which a period of two months was set for correcting certain deficiencies, bore the date of 4 March 2013. This communication was sent by registered letter. However, no evidence of delivery from the postal authorities was available. After examining evidence, the board said that evidence supported the appellant's submissions, according to which the letter was received on 15 March 2013 by a person who was not authorised to collect business mail during its representative's absence, but only private post. This was confirmed by the authorisation filed with the letter dated 7 August 2014. Although this authorisation generally allowed the collection of "all registered mail", it contained an explicit list of several exceptions. These exceptions comprised the collection of mail items which typically related to the business of a professional representative. The collection of letters from patent offices, such as the communication from the EPO at issue, was specifically excluded from the authorisation given. In the board's opinion it had at least been made plausible that the postal authorities erroneously delivered the letter to an unauthorised person in view of an authorisation which seemed to allow the collection of "all registered mail". The appellant also brought evidence as to the absence of its representative from her place of business. The board concluded that, in cases where the EPO bore the burden of proof, the applicant had to be given the benefit of the doubt. If doubts remained about what really happened, this could not be to the detriment of the applicant. This applied all the more in a situation like the one at issue where the refusal of the application was the immediate consequence for the applicant.

F. Suspected partiality

1. Suspected partiality of members of the opposition division objections under Article 24(1) EPC

(CLB, III.J.8.1)

In T 1674/12, opponents 1 and 3 alleged a series of substantial procedural violations, in particular partiality of the opposition division. They inferred partiality from a number of circumstances, one being that a member of the division, namely the primary examiner (i.e. not the chairman), had been involved in granting a divisional patent application resulting from the opposed patent. The opponents maintained that, since this member had granted the divisional application without awaiting the decision on that earlier patent, even though he had been aware of the objections to it, he at least had been biased. The board observed, and the opponents conceded, that, whilst the rules applicable to board members (Art. 24(1) EPC) were different, a member of an examining division which had granted a patent could take part in opposition proceedings concerning the same patent, provided he did not act as chairman (see Art. 19(2) EPC). Such participation was therefore not a sufficient basis for objecting to that member or suspecting him of partiality. The opponents based their allegation that the primary examiner had been biased on the contention that he could not decide on a divisional application before knowing the outcome of the opposition to the earlier patent from which it resulted. Rather, he ought to have taken that outcome into account in deciding on the divisional application. The board observed that the decision to grant a patent was taken by the examining division and not the primary examiner alone. In its view, requiring that the division always await the outcome of a different, albeit related, case would run counter to its independence. Clearly, therefore, the examiner's decision to process the divisional application could not be regarded as a sure sign of partiality. The board observed that, according to case law, partiality could not be established solely on the basis of a party's subjective impression, and that the circumstances also had to be considered from an objective observer's point of view (see T 190/03, OJ 2006, 502, and T 281/03, both of 18 March 2005).

2. Suspected partiality of members of the boards of appeal admissibility under Article 24(3), second sentence, EPC

(CLB, III.J.9.2)

In T 49/11 the board in its original composition had to decide whether the objection was admissible in view of Art. 24(3) EPC. A partiality objection "shall not be admissible if, while being aware of a reason for objection, the party has taken a procedural step" (Art. 24(3), second sentence, EPC). In the case at issue, the admissibility of the partiality objection was disputed because the respondent, after receiving the summons, which made the parties aware of the composition of the board, filed two letters with the board before raising the partiality objection. In the first letter, the respondent, inter alia, expressed its intention to speak German at the oral proceedings. The board held that the statement that a party intends to speak an official language other than the language of the proceedings at oral proceedings constitutes a procedural step ("Verfahrenshandlung", "acte de procédure") within the meaning of Art. 24(3), second sentence, EPC, as it is a formal notification which is required under R. 4(1) EPC. The respondent's statement that it intended to speak German at the oral proceedings was therefore a procedural step, taken more than three months before the respondent raised the partiality objection. The partiality objection was therefore rejected as inadmissible under Art. 24(3), second sentence, EPC. The board declared that, after analysing the text of Art. 24(3), second sentence, EPC in the three official languages (Art. 177(1) EPC) and the difference between Art. 24 EPC 2000 and Art. 24 EPC 1973 and the transitional provisions of the EPC 2000, it would have come to the same result under the old and the new text of Art. 24(3) EPC.

3. Suspected partiality of members of the Enlarged Board of Appeal

(CLB, III.J.10)

In R 19/12 the petitioner, which maintained that its right to be heard had been violated, objected to the chairman of the Enlarged Board of Appeal on suspicion of partiality under Art. 24(3) EPC on account of his involvement, past and present, in the administration of the EPO. In its interlocutory decision of 25 April 2014 on the partiality issue the Enlarged Board stressed that the test for assessing suspected partiality was whether in the light of the facts a reasonable, objective and informed person would have good reason to fear that the judge had not brought or would not bring an impartial mind to bear on the adjudication of the case.

It said the rules on exclusion and objection were designed to maintain the necessary distance for the judge from the issues at stake and the parties thereto and also from the court whose decision was being reviewed. Such distance was especially important where it was a question of the distance of the court and its judges from the administrative authority whose decisions the court was required to review.

The fact that a judge had previously held high office in an administrative hierarchy was not a sufficient ground that on its own justified a suspicion of partiality. In the case in point, however, the judge concerned had been appointed simultaneously as Vice-President DG 3 (VP3) and chairman of the Enlarged Board and remained, as a vice-president, part of the Office's administrative hierarchy. Under Art. 10(2)(f) EPC he remained subject to the supervisory authority of the President as his immediate superior. Under Art. 10(3) EPC the President was assisted by the vice-presidents, this being institutionalised in the form of the MAC (Management Committee) and the GAC (General Advisory Committee; now the General Consultative Committee). VP3 might therefore be faced with conflicting demands. On the one hand, as a vice-president subordinate to the President he had to implement the latter's management and performance objectives, even in relation to the boards of appeal; on the other, as part of his managerial responsibility for the boards he had to ensure that their judicial independence was not prejudiced by measures taken by the President and his administrative hierarchy. That was the position, referred to by the petitioner as a "conflict situation", in which the chairman found himself as a result of his dual function.

There was also a real link between Enlarged Board jurisprudence on the review procedure under Art. 112a EPC and the pursuit of efficiency objectives for the appeal procedure. If the Enlarged Board approved a review of first-instance technical board decisions which was restrictive in terms of the right to be heard, that gave the examining divisions and above all the opposition divisions more latitude to shape proceedings in accordance with prescribed efficiency objectives, e.g. by excluding subsequent requests or submissions.

In view of the Enlarged Board chairman's involvement at senior management level in the Office's administration, there was good reason for a reasonable, objective and informed person to fear that he might not be capable of exercising his judicial function uninfluenced by instructions given to him in his capacity as VP3, and in particular as a member of the above mentioned committees. The Enlarged Board accordingly considered the objection to its chairman to be well-founded.

The interlocutory decision in R 2/14 concerned, inter alia, the petitioner's request that the chairman of the Enlarged Board of Appeal be replaced due to partiality. The petitioner claimed that his judicial function as chairman of the Enlarged Board of Appeal was incompatible with his managerial function as VP3 and referred to the Enlarged Board of Appeal's interlocutory decision in R 19/12, in which the Enlarged Board of Appeal had come to the conclusion that there was a possible conflict of interest between these two functions.

Upon invitation by the Enlarged Board of Appeal, the chairman commented on the petitioner's objections and stated that, following interlocutory decision R 19/12, his managerial activities in the senior management committees of the Office, i.e. the MAC and the GAC, had been discontinued.

Taking into account the chairman's comments, the Enlarged Board of Appeal considered that the present factual circumstances clearly differed from those on which interlocutory decision R 19/12 was based. What remained of the previously established facts was that the chairman, in his managerial function as VP3, continued to be subject to the provisions of Art. 10(2)(f) and (3) EPC, according to which the vice-presidents assist the President and are subject to his supervisory authority. These provisions could come into conflict with Art. 23(3) EPC, according to which the chairman, in his judicial function, is not bound by any instructions and must comply only with the provisions of the EPC. According to the Enlarged Board of Appeal, this "normative conflict" cannot be completely resolved without changes to the current institutional structure of the European Patent Organisation. However, in the meantime, its impact can and must be mitigated by a continuous balancing of these potentially conflicting duties ("normative concordance").

Applying the concept of "normative concordance", the Enlarged Board of Appeal stated that the President's power to give instructions to the chairman in his function as VP3 pursuant to Art. 10(2)(f) and (3) EPC is limited by virtue of Art. 23(3) EPC. The chairman is accordingly relieved of any obligation

(a) to obey any presidential instructions or

(b) to observe other administrative/executive directions or

(c) to assist the President pursuant to Art. 10(3) EPC

if and to the extent that any such instruction, direction or assistance might affect him and/or any other member of the boards of appeal, including the Enlarged Board of Appeal, directly or indirectly, in performing their judicial duties. When the chairman/VP3 is confronted with an unresolvable conflict between a managerial and a judicial obligation, his judicial duties under Art. 23(3) and 24 EPC and under Art. 6(1) ECHR prevail.

In conclusion, the Enlarged Board of Appeal stated that a reasonable, objective and informed person considering the circumstances of the chairman's integration within the Office hierarchy at present would, after the implementation of the institutional measures adopted following interlocutory decision R 19/12, no longer have good reason to suspect the chairman of partiality.

The Enlarged Board of Appeal further held that the factual scope of an objection pursuant to Art. 24(3) EPC is defined in the statement of grounds of objection initiating the interlocutory proceedings under Art. 24(4) EPC. Apart from a subsequent elaboration of said objection by supporting facts, evidence and arguments, the subject-matter of the proceedings, in principle, cannot be extended or changed, whether by new facts or by a new objection. Accordingly, the Enlarged Board of Appeal did not admit a submission in which the petitioner referred to a new category of objections (personal or "subjective partiality" as opposed to structural or "objective partiality"). Moreover, the circumstances on which the petitioner relied had only arisen after the closure of the oral proceedings.

G. Formal aspects of decisions of EPO departments

1. Composition of the competent departments of first instance

1.1 Examining division

(CLB, III.K.2.1)

In T 1207/09 the second member of the examining division present at the oral proceedings was not the same as the second member who signed the summons to those oral proceedings. It was alleged, with reference to T 390/86 (OJ 1989, 30), that this constituted a substantial procedural violation.

The board disagreed, citing T 160/09 with approval. Decision T 390/86 concerned the situation in which the written decision had not been signed by those members of the opposition division who had delivered the decision during oral proceedings. In these circumstances it was questionable whether the written decision reflected the views of the examiners who took the decision in the oral proceedings. T 390/86 did not, however, suggest that the original composition of the opposition division could not be changed during the proceedings up until the oral proceedings. In the present case, the written decision was signed by those examiners who had taken part in the oral proceedings and so T 390/86 was not applicable.

Furthermore, Art. 18(2) EPC set out only that an examining division had to consist of three technically qualified examiners; it did not require that the initial composition be maintained throughout the proceedings.

2. Form of decisions

2.1 Reasons for the decision

2.1.1 Legal requirement for reasoned decisions under Rule 111(2) EPC

(CLB, III.K.4.2.1)

Under Art. 113 (1) EPC, EPO decisions had to show that the parties' fundamental arguments had been addressed (T 2352/13). According to T 246/08: "In brief, a decision must show that all potentially refutative arguments adduced by a party are actually refutable." Merely repeating the parties' submissions was not enough; rather, it had to be clear from the reasons that their core arguments had been addressed in substance in arriving at the decision. In particular, final decisions had to take account of comments received in response to EPO communications (see T 1997/08).

A mere statement might constitute sufficient reasoning if the underlying argumentation of the first-instance department was immediately evident, and the point at issue had not been particularly controversial in the proceedings. But if a party presented detailed arguments, including documentary evidence, about how a term used in the patent would be generally understood by the skilled person, then the examining or opposition division had to take a position on them. In the case at issue, this had not happened, so the right to be heard had been infringed.

2.1.2 Examples of non-compliance with the requirements of Rule 111(2) EPC

(CLB, III.K.4.2.3)

A decision is not reasoned within the meaning of R. 111(2), first sentence, EPC if it is not unambiguously clear from it (possibly after consulting other parts of the file) which request(s) – including any items such as claims, description pages and drawings – it is based on (T 405/12).

H. Correction of errors in decisions Rule 140 EPC

(CLB, III.L.1)

In T 1869/12, the appellant (applicant) lodged an appeal against the decision of the examining division refusing to correct the published European patent specification. Its aim in seeking to correct the decision to grant was that the patent should comprise the set of claims proposed by the examining division with a communication under R. 71(3) EPC, but with substantive amendments as subsequently proposed by the appellant.

The board based its decision on G 1/10 (OJ 2013, 194), which stated that: "Since R. 140 EPC is not available to correct the text of a patent, a patent proprietor's request for such a correction is inadmissible whenever made, including after the initiation of opposition proceedings." Furthermore, the board stated that even if G 1/10 were not taken into account, the appellant's request could not be granted, since it was clear that the examining division had intended – and maintained that intention – to grant the patent with its own proposed claims, not with the claims as requested by the appellant.

While the procedure to grant was defective and there were two procedural errors (R. 111(2), Art. 113(1) EPC), the board stated that the appellant should have filed an appeal against the decision to grant the patent.

The catchword of the decision thus read: In case of procedural violations in the granting procedure, rectification is only possible via appeal against the decision to grant. A correction of the published European patent specification or of the decision to grant cannot be allowed when this specification corresponds to the true intention of the examining division when granting the patent.

I. Stay of proceedings

(CLB, III.M.3)

In J 17/12 the main issue to be decided was whether a transfer of a European patent application should be recorded in the register at a time when proceedings for grant had been suspended under R. 14(1) EPC. The Legal Division held that it should, because registration of the transfer did not affect the grant procedure as such and the register served mainly to inform the public about the current legal status of the application.

The Legal Board of Appeal disagreed. According to the Legal Board the transfer of a European patent application may not be recorded in the European Patent Register at a time when proceedings for the grant of the application are stayed under R. 14(1) EPC. The Legal Board concluded that the registration of a transfer of an application is inconsistent with the fundamental objective of the suspension of the proceedings, which is to protect the third-party claimant's rights in the application. If the registered applicant can be freely changed while proceedings for grant are stayed, the third-party's attempts to obtain the remedies available under Art. 61(1) EPC could be repeatedly frustrated.

There was nothing to stop the filing, during the suspension of grant proceedings, of a request to transfer the application. The effect of a stay in the light of the Legal Board's decision would simply be that no action would be taken on the request during the suspension. While the fact of the transfer might not be apparent from the register, it would be apparent from an inspection of the public file, as would the decision to stay the grant proceedings themselves. The public would therefore be sufficiently informed.

In J 15/13 the appellant (applicant) appealed against the decision of the Legal Division rejecting its request to resume the grant proceedings for its European patent application stayed as from 21 September 2012 pursuant to R. 14(1) EPC, pending a lawsuit filed in Denmark on 20 September 2012.

The appeal was dismissed by the Legal Board of Appeal. It held that the department of first instance had exercised its discretion correctly and that the Legal Division had correctly taken into account all valid interests of the appellant and the respondent.

The Legal Board could not find any evidence that the national proceedings had been filed as a delaying tactic. The filing of the request under R. 14(1) EPC at the last possible moment might only be taken as an argument for the resumption of the grant proceedings if such behaviour appeared to be a misuse of the respondent's right to a stay of the grant proceedings. This was not the case here.

The Legal Board also held that the department of the EPO which had to decide whether to suspend the grant proceedings or not had to verify that the national proceedings were in accordance with R. 14(1) EPC. The Legal Board held inter alia that the national proceedings in question had to be entitlement proceedings not only in name but also in nature. It was acknowledged in the case law of the Legal Board that the EPO was not allowed to examine the substance and merits of a national entitlement case. However, the board's power of examination could not be limited to merely checking whether the claim submitted with the entitlement suit was directed to the transfer of the application but – to a certain extent – also allowed and might even require a consideration of the grounds given in the complaint of the entitlement suit.

The Legal Board came to the conclusion that whether well-founded or not, in the current case, not only the claim but also the arguments brought forward in the national lawsuit were directed to a transfer of the application in suit as a result of such entitlement. Therefore the Danish lawsuit qualified as entitlement proceedings not only in name but also in nature.

Apart from what could be gleaned from the Danish court documents, it could also consider circumstances relating to the conduct of the Danish lawsuit. The Legal Board was aware that the appellant's request for a summary dismissal of the Danish entitlement proceedings had been rejected by the Danish court and that the Danish court was going to hear an expert. Because of these facts, without evaluating the arguments brought forward in the Danish lawsuit, it could be concluded that the Danish court considered the case before it not to be entirely without merit from the outset.

The respondent also stated that its intention was to withdraw the application if it won in the Danish entitlement proceedings. However, this intention did not reduce the respondent's interest in a further stay of the grant proceedings. It was held that even if the entitled proprietor was not interested in the patent, it could be in its interest to prevent the applicant from getting a patent.

J. Rules relating to Fees

1. Ten-day fail-safe arrangement small amounts lacking

(CLB, III.Q.3.1)

In J 25/12 the amount paid for the sixth-year renewal fee and additional fee within the period under R. 51(2) EPC was insufficient due to a prior change in the fee rates. The missing amount was not paid by expiry of the two-month period then set for this purpose (Art. 4(3) of the decision of the Administrative Council of 28 October 2009, OJ 2009, 587) and so the application was deemed to be withdrawn (Art. 86(1), R. 112(1) EPC). Some months later the representative paid the shortfall, making reference to Art. 7(3) and (4) RFees.

The board confirmed the examining division's refusal of the request under Art. 7(3) and (4) RFees, stating that the appellant's interpretation of these provisions appeared to be based on a misunderstanding that payment of a surcharge of 10% created a possibility to pay the missing amount after expiry of the period within which payment should have been made.

The appellant also invoked Art. 8, last sentence, RFees, according to which the EPO may, where this is considered justified, overlook any small amounts lacking without prejudice to the rights of the person making the payment. The board did not doubt that the underpayment here could be regarded as a small amount in relation to the total sum payable. However, even in such a case the EPO could exercise its discretion to overlook small amounts only if that was justified. No such justification existed here, where the representative had been specifically informed about the change in the fee rates and had been invited to pay the amount lacking within a time limit of two months – which however he failed to do.

K. Representation

1. Oral submissions by an accompanying person

(CLB, III.R.5)

In T 8/13 the board stated that the request to allow another person who was not a professional representative to make submissions on behalf of the appellant (opponent) during the oral proceedings was not allowable in view of the conditions set by Art. 134 EPC and having regard to decision G 4/95. In reply to the board's preliminary view expressed in its communication, the appellant argued that Ms L, attending the oral proceedings together with the appellant, had closely followed the whole case from the beginning and should therefore have been allowed to present the case. Moreover, the appellant suffered from asthma and could therefore only plead the case with difficulty. Also, the appellant stated that it was at a disadvantage in view of the fact that the respondent was represented by two professional representatives.

The board was not persuaded by these arguments and refused the appellant's request, as already preliminarily mentioned in its communication. Ms L was neither a representative according to Art. 134 EPC nor was she presented as a technical expert to be heard on a technical question. Her personal involvement with the case or her relationship to the appellant also did not qualify Ms L to have made submissions as an accompanying person in accordance with the conditions set out in G 4/95. In regard to the appellant's medical condition, the appellant was informed by the board that it was prepared to interrupt the proceedings from time to time should this be required.

Whilst the appellant stated that it perceived the procedural situation as unfair in view of the representation afforded by the respondent, the board was nevertheless bound by the relevant provisions of the EPC in this respect. The possibility of representation by a professional representative was of course always open to the appellant. It was, for completeness, added by the board that Ms. L was in the event not prevented by the chairman from assisting the appellant in finding various notes and passages in his submissions and allowing the extra time necessary for this, as this was considered appropriate in the circumstances and was not objected to by the respondent.

In T 1458/11 appellant 2 (patent proprietor) was represented by a professional representative, Mr Ch, accompanied by Mr H, a trainee in the law firm of Mr Ch. Mr H had not been announced prior to the oral proceedings before the opposition division. Appellant 2 requested that Mr H be allowed to speak "on selected issues during the oral proceedings under his (Mr Ch's) supervision and responsibility". Appellant 1 (opponent) agreed to this with the proviso that Mr H would "not make the complete case". The opposition division allowed Mr H to speak "on selected issues under Mr Ch's responsibility". After the impugned decision was pronounced, appellant 1 submitted that, according to its observations, Mr H had spoken "for more than 50% of the time" and that this was not what had been agreed at the beginning of the oral proceedings.

According to G 4/95, the opposition division had the duty to ensure that the oral submissions made by an accompanying person satisfy the conditions that they are made in addition to the complete presentation of the party's case by its professional representative and under the continuing responsibility and control of the professional representative. According to appellant 1, the submissions made by Mr H did not comply with the conditions set forth in G 4/95, i.e. were not in addition to the complete presentation of appellant 2's case by the professional representative, Mr Ch.

The board considered the submissions of the parties and the content of the minutes of the oral proceedings before the opposition division. It was not possible to establish, on an objective basis, whether the submissions made by the accompanying person were in addition to the complete presentation of the professional representative or whether they went beyond that or even amounted to a presentation of substantially the complete case of the party. It was incumbent on appellant 1 to inform the opposition division of any alleged negligence of the opposition division's duty as soon as it became aware of it. This immediate reaction was required since a party to the proceedings must take an active part and must on its own initiative submit in due time whatever will support its position (cf. R 2/08, point 8.5 of the Reasons). In the case at hand, appellant 1 should and could have informed the opposition division accordingly at the latest before the lunch break since, according to its submissions, at that point in time the professional representative had only made a few submissions, i.e. the accompanying person's oral submissions were apparently not only in addition to the complete presentation of appellant 2's case by the professional representative.

Furthermore, nothing indicated that appellant 1 was taken by surprise or that it was not prepared, or at least could not have been prepared, for the oral submissions made by the accompanying person (G 4/95, point 10 of the Reasons, second paragraph). The board failed to see that a procedural violation occurred during the proceedings before the opposition division.

IV. PROCEEDINGS BEFORE THE EPO

A. Preliminary and formalities examination

1. Accordance of a date of filing

1.1 Requirements for according a date of filing

(CLB, IV.A.5.1)

In J 1/12 the appellant had filed a European patent application with the UK Patent Office with the same documents as those underlying the later application before the EPO. The application in the UK never reached the EPO, having been misplaced by the UK Patent Office. The Legal Board therefore had to consider whether, contrary to Art. 80 EPC 1973, it was justified to assign the application the earlier filing date (Art. 75(1)(b) EPC 1973).

Art. 77(2) EPC 1973 provides that European patent applications filed within the member states are to be forwarded to the EPO within six weeks after filing. Art. 77(5) EPC 1973 supplements this provision by providing that European patent applications which do not reach the EPO before the specified time limit are deemed to be withdrawn. This happens irrespective of the reasons why the applications do not reach the EPO. It followed that the application filed with the UK Patent Office was deemed to have been withdrawn. No re-establishment of rights is provided for; re-establishment applies only in situations where there has been a failure to observe a time limit on the part of the user.

In such a situation the legislator expressly provides in the last sentence of Art. 77(5) EPC 1973 for the refund of the filing, search and designation fees. In addition, Art. 135(1)(a) and 136(2) EPC provide facilities for a conversion of the lost European patent application into national patent applications. It followed that the legislator of the EPC clearly recognised the harsh consequences of Art. 77(5) EPC 1973 for an applicant. But, having recognised the problem, the legislator did not provide the applicant with any means to regain the lost application. In J 3/80 it was held that refund options are to be seen as a clear alternative to the missing possibility of restoration of rights. The designation of a filing date belonging to a lost application to the application in suit would thus clearly circumvent the legislative will.

1.2 Corrections under Rule 139 EPC

1.2.1 No replacement of invention

(CLB, IV.A.5.5.2)

In J 16/13 the appellant requested correction of the wrongly filed description and claims under R. 139 EPC. The board did not dispute that the skilled person would have immediately recognised the mismatch between the drawings and the other parts of the description and therefore would also have sought to find out what the correct (or intended) application documents were. However, it was not "immediately evident [for the skilled reader of the application] that nothing else would have been intended than what is offered as the correction", i.e. in this case, the contents of the priority document. A practically infinite set of perfectly plausible replacements could be imagined when a complete description and corresponding claim set turned out to be erroneous. A description completely corresponding to that of the priority document was only one plausible alternative. Furthermore, "common sense" would perhaps lead the skilled person to consult the parallel applications, but it would not provide the firm belief – the immediate evidence – required for R. 139 EPC. This firm belief in the one and only possible correction was also clearly stated in G 3/89.

The board noted that the case before it was similar to J 5/06, which explains that the approach of earlier decision T 726/93, relied on by the appellant, was not applicable following Enlarged Board decisions G 3/89 and G 2/95. Similar considerations as explained in J 5/06 applied to J 4/85, also relied on by the appellant. Thus the reasoning of J 4/85, namely that the intentions of the applicant must be given due consideration when applying R. 88 EPC 1973 (corresponding to R. 139 EPC), had been clearly overruled by later jurisprudence (see J 5/06, point 10 of the Reasons). The request for correction of the specification under R. 139 EPC was refused.

B. Examination procedure

1. Amendments under Rule 137(3) EPC

(CLB, IV.B.2.5.1)

In T 158/12 the appellant's view was that there was no article or rule in the EPC which would prevent the applicant from changing from one invention to another – if they were searched – during examination. The board considered, however, that the articles and rules present in the EPC formed a statutory system which clearly led to the conclusion that there was no provision allowing the payment of multiple examination fees for a patent application. The board stated that only one examination was to be carried out for one application, as only one single examination fee had been paid. Having once chosen one invention (or one group of inventions) to be made the subject of examination, this choice could not be altered once examination of that invention had commenced. The board was of the opinion that this approach based on the statutory provisions of the EPC was confirmed in opinion G 2/92. Hence, the appellant's view that the examination of an application could be based on more than one invention was not supported by this opinion of the Enlarged Board of Appeal.

2. Additional searches during examination

(CLB, IV.B.4.1.2)

In T 2299/10 the decision under appeal was, inter alia, based on the objection that the subject-matter of claim 1 lacked an inventive step. The European Patent Office acting as International Searching Authority issued a declaration of non-establishment of the international search report under Art. 17(2)(a) PCT. No supplementary European search report was established either. The examining division examined the application despite the fact that no search had been carried out. However, this is only possible in exceptional cases and, according to the jurisprudence of the boards of appeal, an additional search for pertinent prior art may be dispensed with only if the technical features of the claims are considered to be "notorious", i.e. generic and so well known that they cannot reasonably be refuted (see T 1411/08, point 4 of the Reasons). In the board's judgment, the technical features went beyond the mere common general knowledge and could not be considered "notorious". An applicant's acknowledgement in the original application that certain prior art was known is in general not a sufficient reason for not carrying out an additional search, since such statements may be – and indeed frequently are – withdrawn or qualified. Moreover, this could only apply in cases where all the technical features in the claim were acknowledged as known (see T 1924/07, point 9 of the Reasons). In the case at issue, however, the appellant did not acknowledge the relevant features of claim 1 mentioned as being known. Thus, claim 1 could not be definitively assessed with respect to novelty and inventive step without knowledge of the relevant documented prior art. Thus the request required a search for relevant prior art. Hence the matter had to be remitted for an additional search and further examination.

3. Amendments relating to unsearched subject-matter

(CLB, IV.B.5.4)

In T 333/10 the decisive question in the case at issue was whether the amended claims according to the appellant's sole request should have been considered admissible under R. 137(4) EPC (version of 13 December 2007). According to R. 137(4) EPC, amended claims may not relate to unsearched subject-matter which does not combine with the originally claimed invention or group of inventions to form a single general inventive concept.

The board found that the subject-matter of amended claim 1 could not be considered to have been searched and it did not combine with any of the originally claimed inventions to form a single general inventive concept. The appellant referred to decision T 2334/11 and argued that, in a case like the present one, no "a posteriori" unity judgment was to be made; rather, it was to be generally examined if the added feature taken from the description combined with the originally claimed invention to form a single general inventive concept focused on in the claims and description as originally filed.

The board noted that the claims as originally filed were found to lack unity by the search division. It also stated that the present case concerned the replacement of some of the features from the combination of features on which the search was based by features which might not be considered as corresponding special technical features within the meaning of R. 44(1) EPC and which could not be expected to constitute a pure limitation of one of the searched inventions. In contrast, decision T 2334/11 concerned a factual situation in which added features restricted the scope of the searched subject-matter. This difference was also addressed in decision T 2334/11, which essentially stated that the jurisprudence concerning R. 137(5) EPC (version of 1 April 2010) distinguished between cases in which the claimed subject-matter was substantially changed, in particular by replacement or omission of a feature in a claim, and which could give rise to an objection under R. 137(5) EPC, and those cases which concerned the pure limitation or concretisation of a claim by adding a feature disclosed in the application as originally filed, and which did not normally cause lack of unity with the originally claimed invention within the meaning of R. 137(5) EPC. In the case before it, the board did not see a limitation or concretisation of any of the four inventions which were initially searched and which could have been pursued by the appellant.

The board noted that decision T 2334/11 could not be understood to the effect that the amendment of an independent claim by a feature extracted from the description is generally admissible under R. 137(4) EPC if the subject-matter of the independent claim has been searched and lacks novelty with respect to a prior-art document. Instead, the board in decision T 2334/11 emphasised that – in such cases – it always has to be examined whether the added feature is linked to the general inventive idea that can be deduced from the claims and description as originally filed. Hence, the amended set of claims was not admissible.

C. Special features of opposition and appeal proceedings

1. Transfer of party status

1.1 Change of address

(CLB, IV.C.2.1)

In T 786/11 the respondent (patent proprietor) requested prior to the oral proceedings that the address of the "applicant/patentee" be changed in two steps, firstly from "INNOVATIVE SONIC LIMITED P.O. Box 957 offshore incorporations centre Road Town, Tortola British Virgin Islands" to "INNOVATIVE SONIC LIMITED 4th Floor Unicom Centre 18N Frere Felix De Valois Street Port Louis Mauritius" and thereafter to "INNOVATIVE SONIC LIMITED 2nd Floor, The Axis, 26 Cybercity, Ebene 72201, Mauritius". The respondent submitted a "certificate of discontinuance" and a "certificate of registration by continuation" from the authorities of the British Virgin Islands and the Republic of Mauritius, respectively, as evidence that the above change constituted a mere change of address.

In response to the notice of the respondent to register a change of its address, the appellant disputed that after moving from the British Virgin Islands to Mauritius the respondent was still the same legal entity as before. The appellant argued that the company "Innovative Sonic Limited incorporated in the British Virgin Islands" had been discontinued and thus ceased to exist. Instead, a new company "Innovative Sonic Limited incorporated in Mauritius" was now replacing the former respondent, and the appellant was not willing to accept that.

The board studied the applicable legal provisions of the British Virgin Islands and of the Republic of Mauritius and noted that these jurisdictions – unlike many others – allow a legal entity to be transferred from one jurisdiction to the other without any effect on the legal entity's identity. The board therefore concluded that the respondent did not change its legal identity and in particular did not cease to exist at any time. Rather, the company was "transferred" from the British Virgin Islands to Mauritius and continued as a legal entity under the laws of Mauritius.

1.2 Opponent status

(CLB, IV.C.2.2)

In T 531/11 the appellant's representative disputed that the original opponent, Höfler Maschinenbau GmbH, still existed and could still be represented, and asked the board for a decision declaring it not to exist. In his submissions, it had been acquired by Klingelnberg GmbH by way of universal succession.

The board pointed out that the excerpt from the purchase contract showed that Höfler Maschinenbau GmbH had been acquired "through sale of individual business assets". The contract's wording showed that the appellant's representative erred in claiming that universal succession had occurred. His argumentation was also flawed because, had universal succession taken place, Klingelnberg GmbH would have automatically taken on opponent status (see T 6/05 and T 425/05) and whether or not the original opponent still existed would no longer have been an issue.

Nor was there any other evidence on file to suggest that the opponent did not exist. So the board saw no reason to doubt its existence, or that its representative was empowered to act on its behalf.

1.3 Evidence

(CLB, IV.C.2.2.6)

In T 2357/12 the patent proprietor questioned the probative force of private documents, as submitted in this case, rather than public registration documents, as evidence of the transfer of opponent status in inter partes proceedings. The board stated that whereas public registers often enjoyed public trust regarding the facts registered and other public documents might be more conclusive on formal questions, the identity of the issuer and the date and place of creation of a private document could be more easily contested. However, neither kind of document provided irrefutable evidence of the correctness of a document's content. Furthermore, the accuracy of public registers was dependent on the accuracy of the information received by the authorities, handed in mostly in the form of private documents.

In most cases, private documents seemed to be the most direct pieces of evidence. Citing examples of the different types of documents accepted in individual cases, the board pointed out that the boards had always accepted public and/or private documents as evidence of the transfer of rights, whether by way of universal succession or a single transfer of assets. The level of proof required was credibility of the facts for which evidence was given, in the light of all circumstances.

2. Withdrawal of opposition

(CLB, IV.C.4.1.2)

In T 46/10 the proprietor had appealed against the opposition division's decision revoking European patent No. 1 488 468 B1, and the respondent (opponent) had withdrawn its opposition.

The board found that by withdrawing its opposition the opponent/respondent had ceased to be a party to the appeal proceedings, as the apportionment of costs under Art. 104 EPC had not been an issue. Furthermore, in keeping with established case law, the withdrawal of the opposition during the appeal proceedings had no immediate procedural significance because the opponent was the respondent and the contested decision had revoked the patent in suit.

In examining the substance of the opposition division's decision the board was able to consider evidence and also arguments submitted by the respondent before the opposition was withdrawn. Ruling that none of the cited grounds for opposition prevented the maintenance of the European patent, it duly set the contested decision aside and rejected the opposition.

3. Apportionment of costs

(CLB, IV.C.7)

In J 22/12 the appeal was directed against what was purported to be a decision of the examining division set out in a letter stating that the appellant's notice of opposition could not be considered as validly filed. The Legal Board of Appeal found that the key question to be decided was whether the examining division's letter was a decision or merely a communication. It was held not to be a decision. Furthermore, it was disputed whether an appeal against a communication issued on behalf of the examining division can be admissible. The board found that this was not the case.

The board has the power to order an apportionment of costs under Art. 104 EPC if it is equitable to do so. Art. 16(1)(e) RPBA refers to abuse of procedure by a party as a circumstance that may justify an apportionment of costs. The board considered that the conduct of the appellant, that is the filing of an opposition and of a subsequent appeal, could not be considered as such an abuse, as they were actions which use the provisions of the EPC for the ends envisaged by these provisions. Thus the board considered it appropriate that in this case each party should bear its own costs.

In T 493/11 the respondent introduced important evidence, i.e. a request for a witness hearing on the relevant alleged public prior use, at a late stage of the proceedings, namely only two months before the date of the scheduled oral proceedings, without cogent reasons for the delay and in particular without submitting all necessary information.

Since the prior use in question had previously been raised and the offer of witnesses had already been made with the opposition, the respondent could have come forward with the above-mentioned supporting submissions at an earlier stage of the opposition proceedings or, at least, directly after receiving the opposition division's summons and communication. If it had submitted the material in question at an earlier date, no postponement of oral proceedings would have been necessary and the appellant's costs would not have been incurred unnecessarily.

It was evident that the postponement of the oral proceedings was done to follow the respondent's submissions and its request for the taking of evidence. Consequently, the fact that the appellant itself had submitted new requests quite late in the appeal proceedings, i.e. on the same day as the respondent had submitted its (incomplete) offer of evidence, obviously was not the decisive factor for the opposition division when cancelling and postponing the oral proceedings.

The board stressed that the responsibility for stating its complete case, including all necessary evidence, and the risk for any failure in doing so lie exclusively within the respondent's own sphere. As a consequence, any delay causing the incurring of additional costs by the other party remains within the respondent's sphere and liability.

It was therefore equitable to order that the respondent meet part of the additional costs incurred by the appellant, i.e. those costs caused by the cancellation of the flight and hotel bookings due to the postponement of the oral proceedings.

In T 1361/09, in the proceedings before the opposition division, both parties had submitted auxiliary requests for oral proceedings. The respondent (proprietor) had requested a different apportionment of costs on the grounds that the appellant (opponent) had not announced that it would not be attending the oral proceedings until shortly before the appointed day. The respondent also argued that the oral proceedings would have been unnecessary given the appellant's non-appearance, but it had itself already travelled to the proceedings because the appellant had withdrawn at such short notice.

According to case law cited by the board, it may be equitable to charge the costs for oral proceedings to a party which withdraws from them at short notice or does not appear at them; but generally the non-appearance of a party does not adversely affect the party which does attend, unless the absence of the first party renders the oral proceedings unnecessary (T 273/07).

In the case in point however the board was not sure that the oral proceedings could have been dispensed with on that account, or that the respondent would not have attended. Both parties had submitted auxiliary requests for oral proceedings, and the summons had been issued without any indication or suggestion that the opposition division would probably find in favour of the respondent. Hence the respondent could not have expected to prevail purely on the basis of its written submissions if it failed to attend. In these particular circumstances it seemed likely to the board that the respondent would have appeared to defend its interests even in the knowledge that the other party had withdrawn.

Thus the costs incurred for the oral proceedings before the opposition division were not necessarily attributable to the conduct of the appellant's representative. Consequently it was not equitable to charge them to the appellant (T 65/05, T 190/06).

D. Opposition procedure

1. Admissibility of opposition

1.1 General

(CLB, IV.D.2.1.1)

The facts underlying the referral to the Enlarged Board of Appeal in case G 1/13 (OJ 2015, A42) were that during the opposition proceedings, the opponent, FBL, a company incorporated under UK law, had ceased to exist under that law. However, its authorised representative had continued with the opposition as if it still existed and had also filed an appeal in its name. In the subsequent course of the appeal, the name of FBL was restored to the UK register of companies by court order and FLB was thereby deemed under UK law to have continued in existence as if it had not been struck off the register. The proprietor (respondent) thereupon requested "revocation" of the opposition division's interlocutory decision on the ground that the opposition proceedings had lapsed when FLB ceased to exist, or, on an auxiliary basis, that the appeal be declared inadmissible (on the latter point, see Chapter IV.E.7.1 "Referral under Article 112 EPC – admissibility").

The Enlarged Board's answer on whether the EPO must recognise the retroactive effect of the relevant provision of national law and allow the opposition to be continued by the restored company was as follows:

"Where an opposition is filed by a company which subsequently, under the relevant national law governing the company, for all purposes ceases to exist, but that company is subsequently restored to existence under a provision of that governing national law, by virtue of which the company is deemed to have continued in existence as if it had not ceased to exist, all these events taking place before a decision of the opposition division maintaining the opposed patent in amended form becomes final, the European Patent Office must recognise the retroactive effect of that provision of national law and allow the opposition proceedings to be continued by the restored company."

The status of an opponent was a purely procedural status (G 3/97, OJ 1999, 245). The right to bring and take part in an opposition and to have a legally binding decision on its requests were matters to be determined by the procedural law of the EPC (T 15/01, OJ 2006, 153). So far as the EPC was concerned, the existence or non-existence of a legal entity was exclusively a matter for national law (ibid.; also G 3/99, OJ 2002, 347). General principles such as equal treatment, legal certainty and procedural efficiency should also be taken into account, as should the interests of the parties and of the general public (T 1091/02, OJ 2005, 14). The conclusion reached by the Enlarged Board was that national law should also be followed as regards the deemed retrospective existence of a legal entity.

The Enlarged Board furthermore made it clear that, in circumstances such as these, full effect should be given to any procedural steps which had been taken while the opponent company was not in existence. The restored company could not be placed in a better position than if it had in fact continued in existence all the while.

1.2 Correction of opponent's name

(CLB, IV.D.2.2.2)

In T 1551/10 the board stressed that for an opposition to be admissible the opponent had to be identifiable by expiry of the opposition period (T 25/85, OJ 1986, 81). Provided that was the case, wrong information could be corrected (T 219/86, OJ 1988, 254; T 870/92).

Extracts from the commercial register showed that Swisscom (Schweiz) AG – at first under its previous name (Swisscom Fixnet AG) – was the (sole) universal successor in title to Swisscom Mobile AG, which was originally named as the opponent, although it had already been removed from the register before the opposition was filed. No-one argued – nor was it otherwise apparent – that any of Swisscom Mobile AG's business assets had been taken over by any other legal entity. Therefore, even though its predecessor in title had been misnamed in the notice of opposition, Swisscom (Schweiz) AG, under its previous or present name, was clearly and unambiguously identifiable as the opponent by the time the opposition period ended.

It was recognised in the case law that procedural statements mistakenly made in the name of a deceased party, or of a legal person that no longer existed following a merger, could be regarded as having been made in the name of their universal successor in title (T 15/01, OJ 2006, 153). The mistake in the opponent's name could therefore be corrected under R. 139 EPC. Nor did the board see any possibility of the public being misled, because the commercial-register extracts showed only one successor in title at the time the opposition was filed, namely Swisscom (Schweiz) AG, under its previous or present name.

Reference is made here to the more recent decision G 1/12 (OJ 2014, A114), in which the Enlarged Board of Appeal held in respect of the wrong identification of the appellant in the notice of appeal that it is possible to correct this error under R. 101(2) EPC and also R. 139, first sentence, EPC (see Chapter IV.E.1.3.1(a) "Rule 99(1)(a) EPC").

In T 1269/11 the board pointed out that both R. 77 and R. 139 EPC were available in principle to rectify errors in the name of the respondent (opponent) as given in the notice of opposition (see G 1/12, OJ 2014, A114).

The issue for a correction under R. 77(2) EPC was whether the board, applying mutatis mutandis the principles of G 1/12, could establish the respondent's true intent with sufficient certainty from the information in the notice of opposition or otherwise on file, i.e. could ascertain which company was to be regarded as being the entity that in all probability had filed the opposition. In the circumstances of the case, however, it was not possible to establish that "Jost-World GmbH" (now "Jost-Werke GmbH") was in all probability to be regarded as this, the opposition having been filed under the incorrect name of "Rockinger GmbH, …". Other alternatives seemed equally if not more likely.

A correction under R. 139 EPC presupposed that the respondent's true intent had always been to act for Jost-World GmbH. As this was not at all obvious, the true intent had to be both set out and proved, and the burden of proof was high (see G 1/12). In the objective circumstances of the case in point, that burden had not been discharged.

2. Scope of examination in the case of amendments

(CLB, IV.D.4.5)

In T 616/12 the board held the opponent-appellant's objections to lack of clarity in amended claim 1 to be inadmissible.

An objection to the clarity of granted claims was not foreseen within the opposition procedure, as lack of clarity was not a ground for opposition. Since claim 1 was a combination of granted claims 1 and 15, a clarity objection to this claim was tantamount to raising an objection of lack of clarity to the claims as granted.

Furthermore, it was not apparent how, nor did the appellant argue why, a lack of clarity resulted from an amendment caused by combining the granted claims. In the granted patent, claim 15 was directly and truly dependent on claim 1 and thus, in accordance with e.g. R. 43(4) EPC, included all the features of that claim. Claim 15 as granted was thus nothing more than an abbreviated form of the combination of features defined in both claims 1 and 15. Since claim 15 was dependent on any of claims 1 to 14, the amendment of claim 1 to include the features of only claims 1 and 15 could not be understood as anything more than a deletion of claim 1 and its substitution by claim 15. The deletion of the back-reference to claim 1 in granted claim 15 could not be regarded as anything but an appropriate and indeed necessary adaptation for reasons of linguistic correctness when moving from the abbreviated form to the full text. Also, the replacement of the words "characterised in that" by "wherein", and the deletion of certain reference numerals which were no longer applicable, were not changes which implied any amendment to the claim in a substantive sense, whereby the clarity of granted claim 15, dependent then on claim 1, also could not be affected.

In T 373/12 (OJ 2014, A115) claim 1 of auxiliary request 1, first submitted during opposition proceedings, consisted of a combination of the features of claim 1 as granted and of dependent claim 3 as granted. Although lack of clarity is not a ground for opposition, amended claims have to meet the requirements of the EPC (Art. 101(3) EPC). As the board considered the jurisprudence of the boards of appeal to be divided on whether the requirements of Art. 84 EPC are to be examined if amended claims are a mere combination of granted claims, the board referred the following questions to the Enlarged Board of Appeal (G 3/14, see below):

1. Is the term "amendments" as used in decision G 9/91 of the Enlarged Board of Appeal (see point 3.2.1 of the Reasons) to be understood as encompassing a literal insertion of (a) elements of dependent claims as granted and/or (b) complete dependent claims as granted into an independent claim, so that opposition divisions and boards of appeal are required by Art. 101(3) EPC always to examine the clarity of independent claims thus amended during the proceedings?

2. If the Enlarged Board of Appeal answers Question 1 in the affirmative, is then an examination of the clarity of the independent claim in such cases limited to the inserted features or may it extend to features already contained in the unamended independent claim?

3. If the Enlarged Board answers Question 1 in the negative, is then an examination of the clarity of independent claims thus amended always excluded?

4. If the Enlarged Board comes to the conclusion that an examination of the clarity of independent claims thus amended is neither always required nor always excluded, what then are the conditions to be applied in deciding whether an examination of clarity comes into question in a given case?

In its decision on referral G 3/14 (OJ 2015, ***), the Enlarged Board first considered its previous jurisprudence, i.e. G 1/91 (OJ 1992, 253), G 9/91 (OJ 1993, 408) and G 10/91 (OJ 1993, 420). It cited point 19 of the Reasons in both G 9/91 and G 10/91: "[...] in case of amendments of the claims or other parts of a patent in the course of opposition or appeal proceedings, such amendments are to be fully examined as to their compatibility with the requirements of the EPC". The Enlarged Board noted that it had not been discussed in these cases whether the term "amendments" had to be understood as being any kind of alteration to a claim, or rather only modifications which are in some way qualitative. Nevertheless, had the Enlarged Board at the time considered that the opposition division and the boards of appeal were given wide powers to examine amended claims, it would have said so (in G 9/91 the claims had been amended before the opposition division). Therefore, the term "amendments" as used above had to be understood in the sense that the subject matter to be examined must have some direct nexus with the amendment.

The Enlarged Board then identified three main lines in the jurisprudence of the technical boards of appeal. T 301/87 and T 227/88 were the origin of the "conventional" approach, which is based on the test whether the amendment introduces any contravention of Art. 84 EPC, and the principle that Art. 101(3) EPC does not allow objections to be based upon Art. 84 EPC if they do not arise out of the amendments made. According to the Enlarged Board, the expression "arise out of" refers to a lack of clarity that did not exist before, so that the amendment gives rise to a lack of clarity for the first time. This approach had been applied in many cases, as "established" case law (e.g. T 381/02, T 1855/07, T 367/96 and T 326/02). Nevertheless, the expression had also been interpreted in a wider sense in subsequent decisions, according to which a lack of clarity could be examined if the amendment "highlights and focuses attention" on a previously existing ambiguity (T 472/88; see also T 681/00 and T 1484/07). The Enlarged Board considered this development of case law not to be legitimate (the test depending on whether a lack of clarity was previously noticed or not, and therefore being arbitrary). The third line of jurisprudence was considered "diverging". According to T 1459/05 (see also T 1440/08 and T 656/07), clarity may be examined on a case-by-case basis as a matter of discretion when the added feature is the only feature that distinguishes the subject-matter of the claim from the prior art. T 459/09 was the most far-reaching of the diverging cases, stating that amended claims may generally be examined for clarity on a case-by-case basis irrespective of the kind of amendment. The Enlarged Board expressed doubts as to the legal basis for such a discretion, which would also mean that parties could hardly predict the outcome of opposition proceedings.

The Enlarged Board then set out how Art. 101(3) EPC was to be interpreted. It pointed out that the requirements of Art. 84 EPC are part of the "requirements of the Convention" for the purposes of Art. 101(3) EPC. There was, however, no indication that the object and purpose of that article was to open up the patent to a complete re-examination, whether for clarity or the other requirements of the EPC. It was rather the amendment itself which was relevant, and its effect for the ground for opposition in question. Self-evidently, the amendment must not itself give rise to new objections.

The Enlarged Board stressed that, as apparent from the travaux préparatoires to the EPC 1973, the legislator had deliberately chosen not to make Art. 84 EPC a ground for opposition. It would be unsatisfactory if an opponent could cause delays whenever an amendment was made, by raising all kinds of Art. 84 EPC objections. Moreover, a lack of clarity could still be relevant in opposition proceedings in that it could influence the decision on issues under Art. 100 EPC (sufficiency, novelty, inventive step), or impact on the grounds of invalidity alleged in national proceedings relating to a European patent. Furthermore, when the EPC 2000 was drafted, no suggestion was made for a change to reverse the effect of the then established case law, i.e. the conventional approach based on T 301/87.

If the deletion of an independent claim with its dependent claims permitted examination of the remaining claims for lack of clarity, Art. 84 EPC would effectively become a ground for opposition in a large number of cases (about 70% of the patents are amended in opposition proceedings), which would be contrary to the legislator's intention. To conclude otherwise for a combination of claims which actually consists of striking out the original independent claim and then writing out the previously dependent claim in full, would be arbitrary and unjustified.

In conclusion, the Enlarged Board approved the conventional line of jurisprudence as exemplified by T 301/87 and disapproved that exemplified by T 472/88 as well as the "diverging" line of jurisprudence. Its answer to the referred questions was:

"In considering whether, for the purposes of Art. 101(3) EPC, a patent as amended meets the requirements of the EPC, the claims of the patent may be examined for compliance with the requirements of Art. 84 EPC only when, and then only to the extent that, the amendment introduces non-compliance with Art 84 EPC."

3. Correction of error (in claim) in opposition

The catchword of decision T 657/11 reads:

"In opposition proceedings mistakes or errors concerning the claims, the description or the drawings of the patent as granted may be removed either by an amendment occasioned by a ground of opposition under Art. 100 EPC or, as far as the mistakes or errors concern texts or drawings which remain unamended, by way of a correction pursuant to R. 139 EPC."

The claims according to the main request before the opposition division and the appellant's initial request in the appeal proceedings differed from the granted claims only in that the wording "NF permeate" in the last process step of claim 6 was replaced by "NF concentrate". The board had communicated to the appellant that its request was actually for correction of an error in the granted version of the patent, which would have been a matter of R. 140 EPC in that it related to a purported mistake in a decision of the Office, namely the decision to grant the patent. However, R. 140 EPC was not available to correct the text of a granted patent (G 1/10, OJ 2013, 194).

The claims according to the appellant's final (sole) request were, as compared to the claims as granted, amended beyond the mere removal of an error, namely by limiting them to those granted as (process) claims 6 to 11. Hence the basis for the decision on the appeal (and, thereby, on the opposition) was no longer that for the decision to grant, which would definitely lose its effect and be replaced by a new decision. In such a situation any (further) amendment of the claims, even if aimed at removing an obvious mistake in the claims as granted, did not constitute a correction of an error in a decision of the EPO within the meaning of R. 140 EPC.

As pointed out in G 1/10, it was always open to a patent proprietor to seek to amend his patent during opposition or limitation proceedings and such amendment could remove a perceived error. However, an amendment with the (sole) aim of removing a mistake or error in the claims, but also in the description and drawings of the patent as granted, could not be said to be "occasioned by a ground for opposition under Art. 100" (R. 80 EPC). It followed that such mistakes or errors – here the expression "[nanofiltration] permeate"- in the unamended part of the text could only be removed by a correction pursuant to R. 139 EPC, which provision and the specific requirements defined therein applied independently of R. 80 EPC. The proposed correction was in accordance with those requirements.

4. Late-filed facts and evidence in opposition proceedings

4.1 Opposition divisions' exercise of discretion

(CLB, IV.C.1.3.3)

With regard to the opposition division's decision not to admit late-filed documents, the board in T 544/12 recalled that such a discretionary decision should only be overruled in appeal if the discretion has been exercised according to the wrong principles, or without taking into account the right principles, or in an unreasonable way (G 7/93, OJ 1994, 775; T 1119/05). A bare assertion of lack of prima facie relevance was not by itself sufficient reasoning. Without any sufficient reasons being given by the opposition division for not admitting the late-filed documents, the board was not in a position to decide whether or not the opposition division had exercised its discretion in an appropriate way.

In such a situation, it was first necessary for the board to put itself in the place of the opposition division and to decide whether or not it would have exercised such discretion in the same way as the opposition division did (if the board came to the conclusion that, in the position of the opposition division, it would not have exercised its discretion to admit these documents, the separate question would then arise whether the board should exercise its discretion to admit the documents as documents (late-)filed in the appeal proceedings). In the event, the board considered that the documents had been prima facie relevant and should have been admitted. It accordingly overturned the decision of the opposition division and admitted them.

5. Fresh grounds for opposition

(CLB, IV.D.5.3)

In the decision under appeal in case T 2449/12, the opposition division did not deal with the merits of a ground for opposition which had been both raised for the first time and withdrawn during the oral proceedings before it. According to the appellant (opponent), the fact that the opposition division decided not to raise this ground of its own motion was a wrong exercise of its discretion, and this discretionary decision should be overturned, thus introducing this ground of opposition into the proceedings.

The board pointed out that when neither the merits nor the admittance of a ground for opposition which was late-raised and withdrawn during the opposition proceedings were addressed in the decision under appeal, this ground is considered a fresh ground for opposition which cannot be introduced into the appeal proceedings without the consent of the patent proprietor. The fact that the opposition division did not raise this ground of its own motion did not imply that it took a decision – subject to review of the board of appeal – to disregard it, as might have been the case if this late-filed ground had been maintained. Taking a different position on this point and accepting that all the grounds that have not been introduced ex officio have been the subject of a discretionary decision to disregard them would open the possibility to circumvent the principle laid down in decision G 9/91 that fresh grounds for opposition may not be introduced at the appeal stage without the consent of the patent proprietor.

The ground for opposition in question was to be considered a fresh ground of opposition in the sense of decision G 9/91. As the consent of the patent proprietor had been denied, this ground had to be disregarded.

E. Appeal procedure

1. Filing and admissibility of the appeal

(CLB, IV.E.2)

In T 895/13 the board confirmed that, under the notice from the Vice-President Directorate-General 3 dated 17 March 2008 (OJ 2008, 220), acceleration is always a matter for the exercise of the board's discretion. While trivial reasons would clearly not warrant acceleration, there is no fixed standard of proof. Acceleration is not limited to the examples given in the notice, but is a matter to be decided in the discretion of the board on the particular facts of the case before it.

It was clear in the case at issue that the board could have accelerated the procedure ex officio under the notice, in view of the disadvantage which could ensue before the Belgian courts from the suspensive effect of the appeal in question (on application by patentees for preliminary measures, the Belgian courts do not consider the validity of patents even in cases where a revocation decision is suspended pending appeal). If the board could allow acceleration on that basis of its own motion, it could clearly do so on the request of a party (in this case the respondent (opponent)).

Nor was this contrary to the principle of equal treatment; the approach of the Belgian courts in itself posed a disadvantage to Belgian opponents compared to others, and in the case at issue it was the combination of the approach of the Belgian courts together with the appellant's prior conduct on which the respondent relied. Further, the appellant had declined to give a clear and unequivocal statement of intent not to commence proceedings in Belgium.

The board ordered the appeal proceedings to be accelerated. It added that its views related only to the particular facts of this case and the co-pending appeal in T 1125/13 between the same parties, and were not intended to set any precedent for application beyond those facts.

1.1 Appealable decisions

(CLB, IV.E.2.2)

1.1.1 Decisions

a) Examples where the communication was not an appealable decision

In J 22/12 the appeal was against what purported to be a decision of the examining division stating that the appellant's notice of opposition could not be considered as validly filed. The background to this decision involved two appeals: the appeal which was the subject of this decision, and an earlier appeal filed by the proprietor against the grant of its patent. Concerning the latter, the examining division had informed the proprietor's representative that it would rectify its decision to grant the patent. The appellant was then informed that, as the decision to grant had been rectified, the examination procedure was resumed, and it was not possible to file an opposition during this part of the procedure.

The key question to be decided was whether the examining division's letter was a decision or a communication. This depended on its substance and not on its form.

A valid appeal against the decision to grant had been filed by the respondent (patent proprietor) after the publication of the mention of the grant. Art. 106(1) EPC provides that an appeal has suspensive effect. Thus, by filing a (valid) appeal against a decision to grant, an applicant (patent proprietor) prevents the decision to grant from having any legal effect until the appeal is resolved. On the other hand, even after an appeal the decision as such remains and can only be set aside or confirmed by the board of appeal or by way of interlocutory revision under Art. 109(1) EPC. This means that the validity of an opposition filed against the patent concerned depends on the outcome of the appeal of the applicant (patent proprietor) against grant.

Thus, by the time the appellant filed its notice of opposition, the legal consequences of the decision to grant the patent had become subject to the suspensive effect of the respondent's (patent proprietor's) appeal against the decision to grant. As the examining division decided to rectify its decision to grant under Art. 109 EPC, neither this decision nor the publication of mention of grant had any legal consequences. Thus no granted patent was deemed to have existed against which an opposition might have been filed.

The letter was therefore to be seen as a communication. Art. 107 EPC provides that any party to proceedings adversely affected by a decision may appeal. As the letter sent did not constitute a decision, the appellant was not a person entitled to appeal under Art. 107 EPC. The appeal was rejected as inadmissible.

1.2 Board competent to hear a case

(CLB, IV.E.2.3)

In G 1/11 (OJ 2014, A122) the Enlarged Board of Appeal held that a technical board of appeal was competent to hear an appeal against an EPO examining division's decision – taken separately from its decision granting a patent or refusing the application – not to refund search fees under R. 64(2) EPC.

In the case underlying J 16/13, the question arose whether the board competent for deciding on the appeal under Art. 21(3) EPC should be chosen according to the (erroneous) formal legal effect as derivable from the cover page of the decision or the Register, or according to the intended (factual) legal effect as derivable from the body of the decision.

The board disagreed with T 1382/08, where the board derived the competence of a technical board under Art. 21(3)(a) EPC 1973 from the formal legal effect of the decision (refusal), in spite of the fact that it was obviously erroneous. This approach had certain disadvantages; a likely remittal to the department of first instance on the basis of a substantial procedural violation, or a "formally competent" board taking up the case might choose to decide the case on the merits, but the case would be decided by a board which in fact was not competent, given the legislative intent behind Art. 21(3) EPC.

The board therefore held that, where the formal order of an appealed decision is obviously erroneous to such an extent that it is essentially incomprehensible when compared with the reasons in the body of the decision, in particular when it plainly does not correspond to any of the foreseen possible legal effects flowing from the substantive issue underlying the appealed decision, and if the competence pursuant to Art. 21(3) EPC is supposed to change from one board to another only because of such an "impossible" order, the formal order can be disregarded for the purposes of Art. 21(3)(a) and (c) EPC, and rather the substantive request underlying the decision must be guiding in determining the board competent to decide the case (T 1382/08 not followed).

See also Chapter IV.A.1.2 "Corrections under Rule 139 EPC".

1.3 Form and time limit of appeal

1.3.1 Form and content of notice of appeal (Rule 99(1) EPC)

(CLB, IV.E.2.5.2)

In the case underlying G 1/13 (OJ 2015, A42) an opposition had been filed by a company which subsequently, under the relevant national law governing the company, for all purposes had ceased to exist. However, the company was subsequently restored to existence under a provision of that governing national law, by virtue of which the company was deemed to have continued in existence as if it had not ceased to exist.

The Enlarged Board held that the EPO had to recognise the retroactive effect of that provision of national law. Thus, where a valid appeal was filed in due time in the name of the non-existent opponent and the restoration of the company to existence with retroactive effect took place after the expiry of the time limit for filing the notice of appeal under Art. 108 EPC, the board of appeal had to treat the appeal as admissible.

The Enlarged Board referred to the clearly established principle under the EPC that national law should be referred to in order to determine whether a legal entity existed or had ceased to exist, and had capacity to act. The Enlarged Board also considered that the EPO should follow national law as regards the deemed retrospective existence of such a legal entity. This was merely to apply the general principle that such issues are the exclusive concern of national law.

a) Rule 99(1)(a) EPC

In G 1/12 (OJ 2014, A114) the Enlarged Board of Appeal reformulated the question originally posed to ask whether, when a notice of appeal, in compliance with R. 99(1)(a) EPC, contains the name and the address of the appellant as provided in R. 41(2)(c) EPC and it is alleged that the identification is wrong due to an error, the true intention having been to file on behalf of the legal person which should have filed the appeal, it is possible to correct this error under R. 101(2) EPC by a request for substitution by the name of the true appellant.

The Enlarged Board answered that it was possible, provided the requirements of R. 101(1) EPC had been met. Given the explicit reference in R. 101(1) EPC to Art. 107 EPC and the possibility of remedying deficiencies only within the two-month appeal period under Art. 108, first sentence, EPC, the identity of the appellant, i.e. the person entitled to appeal, had to be established by expiry of the two-month period prescribed in Art. 108, first sentence, EPC at the latest.

R. 101(2) EPC concerns deficiencies concerning the indication of the appellant's name and address as prescribed by R. 99(1)(a) EPC. They may be remedied irrespective of the time limits pursuant to Art. 108 EPC, upon invitation by the board of appeal.

The Enlarged Board fully endorsed the case law, in which it was considered that an incorrect indication of the appellant's identity is a deficiency which can be remedied, provided "its correction does not reflect a later change of mind as to whom the appellant should be, but on the contrary only expresses what was intended when filing the appeal" (see T 97/98).

The principle of the free evaluation of evidence would be contradicted by laying down firm rules of evidence defining the extent to which certain types of evidence were, or were not, convincing. This also applied to the problems under consideration in the referral at issue, (referring to G 3/97 (OJ 1999, 245) and G 4/97 (OJ 1999, 270)).

The Enlarged Board also found that in cases of an error in the appellant's name, the general procedure for correcting errors under R. 139, first sentence, EPC was available under the conditions established by the case law of the boards of appeal. The following principles have been established:

(1) The correction must introduce what was originally intended. It is the party's actual rather than ostensible intention which must be considered.

(2) Where the original intention is not immediately apparent, the requester bears the burden of proof, which must be a heavy one (J 8/80).

(3) The error to be remedied may be an incorrect statement or an omission.

(4) The request for correction must be filed without delay.

Furthermore, an allowable correction under R. 139 EPC has retrospective effect (J 4/85 as endorsed in several subsequent decisions, for example J 2/92, OJ 1994, 375; J 27/96; J 6/02; J 23/03 and J 19/03).

See also T 2045/09 and Chapter IV.E.7.1 "Referral under Article 112 EPC".

b) Rule 99(1)(c) EPC

In T 620/13 the notice of appeal did not include anything that the board could recognise as an indication of the decision impugned, nor did it contain or a request defining the subject of the appeal. The board pointed out that the notice of appeal is a legal declaration or legal statement made by a party to a proceeding before the EPO, and as such it must contain an unambiguous, clear and most of all, explicit statement, which is recognisable as a legal statement, concerning both the identification of the impugned decision and the subject of the appeal (see also J 19/90). It was immaterial that the registrar was able to establish which decision was appealed, so that he was also able to inform the parties of the commencement of an appeal and the allocated appeal number.

Concerning the required request defining the subject of the appeal, the board recognised that such a request might be implicit. However, according to the correct reading of T 358/08, what might be "implicit" was that part of the request which indicates whether the decision is to be set aside in whole or only in part. Decision T 358/08 lent no authority to the argument that there need not be any request at all.

The appeal was rejected as inadmissible.

1.4 Statement of grounds of appeal

1.4.1 Form of statement of grounds

(CLB, IV.E.2.6.2)

The board in T 74/12 pointed out that R. 99(2) EPC did not require a case number to be indicated in the statement of grounds of appeal. That logically implied that an appeal could not be found inadmissible simply because the case number was missing, as only the law, viz. the EPC and its implementing regulations, could define the conditions for taking certain procedural steps and the consequences to be expected if those steps were not taken. A procedural principle stated that rights or a means of redress could be lost only if such a consequence was clearly and precisely prescribed.

1.4.2 General principles

(CLB, IV.E.2.6.3)

a) Grounds to be analysed vis-à-vis reasons in decision

In the case at issue in T 501/09, the statement of grounds of appeal of the appellant (opponent) relied on documents which were not part of the first-instance opposition proceedings but were filed by the appellant with its statement of the grounds of appeal. Its argumentation on appeal was thus based on new evidence instead of providing reasons why the conclusions of the opposition division were considered incorrect.

The board was aware that other boards of appeal had found that an appeal based entirely on new evidence may be admissible when the grounds for opposition have remained the same. However, if this new evidence is subsequently not admitted into the appeal proceedings, this means that the appellant's case on appeal is not substantiated (see T 1557/05) and the appeal is unallowable.

The board also pointed out that the purpose of the inter partes appeal procedure is mainly to give the losing party a possibility to challenge the decision of the opposition division on its merits. The appeal proceedings are thus largely determined by the factual and legal scope of the preceding opposition proceedings (see G 9/91 and G 10/91, OJ 1993, 408 and 420). The parties thus have only limited scope to amend the subject of the dispute in second-instance proceedings and this principle is reflected in Art. 12(4) RPBA.

The board decided not to admit the fresh evidence and concluded that the appeal was not substantiated, contrary to the requirements of Art. 108 EPC in conjunction with R. 99(2) EPC. The appeal was rejected as inadmissible.

Similarly, in T 727/09 the statement of grounds set out a reasoned argumentation as to why the patent should be revoked for lack of inventive step based exclusively on newly filed documents, without any reference to the reasons for the impugned decision, let alone any explanation as to why this decision was incorrect. There was thus an entirely new set of facts, albeit based on the same ground for opposition. The board concluded that the aim of the appeal was not to challenge that decision, but to file a second opposition against the patent, the first having failed. The EPC does not allow such a broad interpretation of the aim of an appeal following opposition proceedings.

The requirements for filing an admissible appeal being substantially the same as the requirements for filing an opposition, the same interpretation should apply. The statement of grounds required by R. 99(2) is not a simple formal request for a general re-examination of the case by a second-instance body but a reasoned statement in the framework of what was the legal and factual subject-matter of the decision impugned (see G 9/91).

The board was aware that the criterion of a statement of grounds able to overcome the reasons for the adverse decision had been used in a significant body of case law where the statements of grounds were regarded as sufficiently reasoned to render the appeals admissible in so-called "fresh cases". Nevertheless, a direct and clear link with the impugned decision had to be maintained. As this was not the case here, the appeal was rejected as inadmissible.

However, in T 1915/09 the board found the appeal admissible. The statement setting out the grounds for appeal indicated in detail why the appellant considered that the subject-matter of the claims intended to be maintained by the opposition division lacked an inventive step in view of document (D1) alone (cited during the opposition proceedings) or its combination with, inter alia, document (D6) (cited during the appeal proceedings). The statement thus gave detailed reasons why the decision under appeal should be set aside and indicated the facts and evidence in support of the respective arguments.

The board held that R. 99(2) EPC does not exclude that such evidence is submitted for the first time in appeal proceedings, nor does it require that this evidence is admitted into the proceedings (following T 389/95). Therefore, the fact that the appellant relied inter alia on documents (D6) to (D13), cited for the first time during the appeal proceedings, in its statement setting out the grounds for appeal did not contravene the requirements of R. 99(2) EPC.

In T 899/13 the board held that one situation in which the requirement for a sufficient statement of grounds in accordance with Art. 108, third sentence, and R. 99(2) EPC was not met was where there had been several independent reasons for the decision to refuse the application and at least one of those reasons was not addressed in the statement of grounds at all or only in insufficient detail. If the statement of grounds for an appeal against the refusal of an application did not address all the stated reasons for that refusal, either by giving sufficient reasoning or through the filing of amended claims, the board could not normally reverse the contested decision, even if it concurred with the appellant in relation to all the reasons for refusal addressed in the statement of grounds. No other outcome was conceivable, save in exceptional circumstances.

However, the case underlying T 395/13 was different. Although the statement of grounds of appeal contained no substantive arguments addressing the reasoning of the decision under appeal, it nonetheless met the requirements of Art. 108 EPC in combination with R. 99(2) EPC. This was because the main objection of the appellant in the grounds of appeal was that the decision under appeal did not contain sufficient reasoning to enable the appellant to file such arguments. The grounds of appeal contained reasoning as to why the appellant considered the decision under appeal to be deficient in this manner, and hence why that decision should be set aside. R. 99(2) EPC required no more than this, and in particular did not specifically require that the statement of grounds of appeal addresses the arguments in the decision under appeal. The board therefore concluded that the appeal was admissible.

1.4.3 New case raised by opponent (appellant)

(CLB, IV.E.2.6.5)

In T 27/13 the board held that the objection under Art. 83 EPC in the statement of grounds constituted a fresh ground for opposition under Art. 100(b) EPC that was outside the legal framework of the opposition proceedings, which were limited to lack of inventive step under Art. 100(a) EPC.

T 389/95 as cited in T 1007/95 found in an obiter dictum that it followed from G 10/91 – which allowed even fresh grounds for opposition to be considered in appeal proceedings if the patentee approved – that an appeal based solely on such grounds was not ipso facto inadmissible.

However, in the light of G 10/91's detailed observations on the legal character of opposition and opposition appeal proceedings and the particular importance it attached to grounds for opposition in that they established the legal framework within which substantive examination of the opposition was in principle to be conducted, the board did point out that it could find the present appeal admissible only if the same ground for opposition was raised when the appeal was filed. Only then could there be any discussion of the possibility of introducing a fresh, additional ground for opposition.

The board therefore took the view that the above-mentioned finding in T 389/95 lacked any basis in G 10/91, which to the board's mind required the appeal to have already been admissibly filed, such that it was not applicable to cases like the present in which the appeal was based solely on an entirely new ground for opposition. As the above finding had not been reflected in subsequent relevant case law even though T 389/95 had been cited quite frequently, in relation to the case in point the board regarded it as an isolated decision that should not be followed, and it followed T 1007/95 instead.

Hence the appeal was inadmissible.

2. Substantive examination of the appeal

2.1 Binding effect of requests no reformatio in peius

2.1.1 Opponent as sole appellant

(CLB, IV.E.3.1(f))

The board observed in T 111/10 that the exceptional possibilities for amendment foreseen by G 1/99 in application of the principle of the prohibition of reformatio in peius only applied if the patent would otherwise have to be revoked, i.e. provided that, in view of the prohibition of reformatio in peius, the patent proprietor (respondent) did not have any other possibility of amendment which would allow rescue of even part of the opposed patent. In the case at issue, however, such a possibility existed. Although this would have resulted in a restriction of scope, it could not be considered unduly inequitable for a patent proprietor who, not having appealed, could not from the outset expect to get more than what was already maintained by the opposition division. It was clear from G 1/99 that any exception to the principle should be construed narrowly.

Decision T 1979/11 was not relevant as in the underlying case an amendment restricting rather than broadening the scope was apparently not available. Moreover, the broadening of scope was a reaction by the respondent/patentee to an objection by the appellant/opponent which had been raised for the first time in the statement of the grounds of appeal; the board in T 1979/11 thus concluded that it would not be equitable to allow the appellant/opponent to present a new attack and at the same time to deprive the proprietor/respondent of a means of defence. In the present case, however, there had been no new attack by the appellant but rather an objection raised by the board and then also taken up by the appellant.

The request was therefore not admitted.

2.2 Patentability requirements under examination in ex parte proceedings

2.2.1 Ex parte proceedings

(CLB, IV.E.3.3.1)

In the case at issue in T 1367/09, when preparing the communication pursuant to Art. 15(1) RPBA the board did not raise the issue of Art. 84 EPC 1973. Nevertheless, when reconsidering the case in preparation for the oral proceedings the board noted that Art. 84 EPC 1973 should indeed be addressed in the oral proceedings. The board noted that in the light of G 10/93 (OJ 1995, 172) the board had the power, in ex parte appeal proceedings, to enforce any grounds for refusal of an application, in particular grounds which the examining division did not consider at all or held to be satisfied. See also Chapter III.B.1 "Non-attendance at oral proceedings".

2.3 Facts under examination applying Article 114 EPC in appeal proceedings

2.3.1 Boards to examine facts of own motion

(CLB, IV.E.3.4.1)

In T 1574/11 the board observed that the principle of examination by the EPO of its own motion (Art. 114 EPC) also applied to appeal proceedings. Hence it was not restricted to the facts, evidence or arguments provided by the parties. As the primary purpose of the appeal procedure was to check that the contested decision was correct, the board was in principle entitled to consider the evidence provided therein if it deemed it to be relevant to its own decision. It observed that the contested decision had also taken E6 into account, and therefore that citation was a part of the contested decision; but the matters in dispute were unchanged.

It followed that the board was entitled to take E6 into consideration in the appeal proceedings.

3. Remittal to the department of first instance

3.1 General

(CLB, IV.E.7.1)

In the decision under appeal in T 1343/12, the examining division found that the claimed subject-matter lacked inventive step, considering document D1 to represent the closest state of the art. A machine translation into English of this document was provided. However, due to the poor quality of the machine translation no unambiguous information could be gathered therefrom regarding one of the features of claim 1 at issue (concentration and effects of a surfactant component). Since it appeared that the assessment of inventive step hinged to a great extent on the true disclosure of D1 in this respect, the case was remitted to the examining division for re-consideration of the issue of inventive step in the light of a certified translation thereof.

3.2 Remittal following filing or late submission of relevant new document

3.2.1 Ex parte appeal proceedings

(CLB, IV.E.7.2.2)

The board in T 963/09 had raised certain objections to the application based on a document it had introduced. The appellant requested that the case be immediately remitted to the examining division for further prosecution. The board refused. In G 10/93 the Enlarged Board had decided that a board of appeal in ex parte appeal proceedings had the power to raise new objections and to introduce new documents, stating that this did not mean that boards of appeal carried out a full examination of the application as to patentability requirements. If, however, there was reason to believe that a condition for patentability might not have been satisfied, the board either incorporated it into the appeal proceedings or ensured by way of referral to the examining division that it was included when examination was resumed.

First, it was clear that an objection raised by a board in the form of a non-binding preliminary opinion did not bind that board and so did not bind an examining division to which the case was later remitted. Secondly, if the case was eventually remitted to the examining division with the instruction to examine the new objection, it might be necessary to include an indication of the relevant grounds, facts and evidence to be considered. Thirdly, where a board of appeal was willing to decide on a new objection itself, it was only fair for the board to fully disclose its concerns to the appellant in order to give it the best possible opportunity to either convince the board that the case should be remitted or argue against the new objection.

4. Late submission in appeal proceedings

4.1 Article 12(4) RPBA submission of new arguments on appeal

(CLB, IV.C.1.4.4)

In T 607/10, in the reply to the statement of grounds of appeal the opponent argued that the subject-matter of claim 1 of the main request did not involve an inventive step having regard to document E19 (seen as the closest prior art) in combination with document E26. At oral proceedings before the board the opponent stated that document E19 was not to be regarded as the closest prior art, and presented a different argument based on document E26 (seen as the closest prior art) in combination with document E19.

The board considered that, in deciding whether a new argument has the effect of amending a party's case within the meaning of Art. 13(1) RPBA, it must be established on a case-by-case basis whether the new argument is a departure from, or just a development of, the original arguments filed with the grounds of appeal or the reply thereto.

In the case at hand, the new argument could not be seen as merely a further development or elaboration of the opponent's previous position. The new analysis was based on a different choice of the closest prior art (document E26 instead of E19), and this in turn meant that, compared to the previous argument, it was necessary to consider a different set of distinguishing features, a different objective problem to be solved and different reasons (based now on document E19) why the skilled person would find it obvious to arrive at the distinguishing features having regard to the prior art. This new argument had therefore to be seen as constituting an amendment to the opponent's case within the meaning of Art. 13(1) RPBA, which might be admitted and considered only at the discretion of the board.

A further complicating factor was that the proprietor, although duly summoned, was not present at the oral proceedings. The board referred to T 1621/09 according to which it remains a matter for the board's discretion to allow an amendment to a party's case in the absence of the prejudiced party. The absence of the prejudiced party is a factor to be taken into account but does not prevent the board from allowing the amendment and proceeding to reach a decision on the basis of the case as now amended. In the case at hand, the board, in the exercise of its discretion, decided to admit the new arguments put forward by the opponent. See also T 1941/10 and T 216/10, where the late-filed arguments were admitted into the proceedings.

4.2 Submission of new documents

4.2.1 Justified reaction to the opposition division's decision

(CLB, IV.C.1.4.5 b))

In T 241/10 the board stated that the board has no power under Art. 12(4) RPBA to hold a document filed with the statement of grounds of appeal inadmissible if the filing of that document was a legitimate reaction to the submission of amended claims by the patent proprietor shortly before the first-instance oral proceedings and the opponent could not have been reasonably expected to present that document in the proceedings before the opposition division (points 2 to 7 of the Reasons).

4.2.2 Late submission of public prior use

(CLB, IV.C.1.4.6 d))

In T 691/12 the board observed that a prior-use allegation first submitted in the appeal proceedings could not be admitted and considered as prior art unless it met at least the following three conditions:

(a) it did not involve an evident abuse of procedure;

(b) the prior use as alleged had prima facie to be so relevant that it cast doubt on the validity of the patent; and

(c) the prior use had to be fully proven, requiring no further investigation to establish its nature or context.

In the case in point these conditions were not met. Therefore, exercising its discretion under Art. 114(2) EPC and Art. 12(4) RPBA, the board concluded that the new challenge, being based on public prior use first alleged in the statement of grounds of appeal, should not be considered.

Likewise in T 444/09 the prior-use allegation first submitted in the statement of grounds was not considered prima facie relevant and was therefore not admitted.

5. Filing of amended claims in appeal proceedings

5.1 Amendments after arrangement of the oral proceedings

(CLB, IV.E.4.2.2)

In T 1732/10 the respondent (patent proprietor) did not file any reply to the appeals within the time limit of four months from receipt of the statements of grounds of appeal. In the course of the appeal proceedings, the board provided its preliminary non-binding opinion that the claims of the patent as granted did not appear to fulfil the requirements of the EPC in view of Art. 54(1) and 56 EPC. In reaction, the respondent filed a new main request together with auxiliary requests 1 to 5. The substantiation for these requests with respect to novelty and inventive step came only later together with additional auxiliary requests 6 to 10. During the oral proceedings admissibility of the respondent's requests in the light of the provisions of Art. 12(1), 12(4), 13(1) and 13(3) RPBA was discussed.

The board stated in its catchword that not reacting in substance to the appeal of the opponent, but waiting for the board's preliminary opinion before any substantive reaction is filed, is regarded as an abuse of procedure. It is contrary to the equal distribution of rights and obligations upon both sides in inter partes proceedings and to the principle that both sides should set out their complete case at the outset of the proceedings. Both principles are clearly established by the Rules of Procedure of the Boards of Appeal.

This is all the more so if the substantiation for all the requests, which were filed after summons to oral proceedings have been sent, is filed only shortly before the oral proceedings before the board. Such requests – which are not self-explanatory – are considered by the board as submitted only on the date of their substantiation. Such very late requests are contrary to procedural economy, do not take account of the state of the proceedings and cannot be reasonably dealt with by the board without adjournment of the proceedings or remittal to the department of first instance, contrary to Art. 13(1) and 13(3) RPBA.

In T 1836/12 the board held that filing new requests without addressing all the points raised in the annex to the summons to oral proceedings made the procedure inefficient, contrary to the principle of procedural economy. Simply filing unsubstantiated requests did not overcome any objections that had already been raised by the board in the annex to the summons or that were to be expected in the light of the contested decision. Given the lack of substantiation, the board had informed the appellant well in advance that the admissibility of these new requests would have to be discussed at the oral proceedings and that they would be considered filed only on the date of their substantiation (T 1732/10). Yet the appellant had provided no substantiation that the claimed new subject-matter was patentable, merely informing the board that it would not be represented at the oral proceedings, which were duly held in its absence. Consequently the board, exercising its discretion under Art. 13(1) RPBA, decided not to admit the requests into the proceedings.

5.2 Article 12(4) RPBA requests not examined at first instance

5.2.1 General principles

(CLB, IV.E.4.3.1)

In T 419/12 the board observed that refusing under Art. 12(4) RPBA to admit a request first submitted in appeal proceedings entailed indicating a specific reason why that particular request not only could but should have been submitted at first instance.

One such reason might be that a corresponding request had already formed part of the proceedings but no decision on it had been sought (T 495/10) or that its drafting and filing had been an obvious step because it clearly could have remedied a deficiency in the requests on file which had been a focus of debate between the parties (T 339/06) and/or the subject of a first-instance communication (T 379/09). The patent proprietor was not at liberty to prevent its case being settled at first instance and shift it to the second instance as it saw fit ("forum shopping", T 1067/08).

The board found that what all these scenarios had in common was that the proprietor had had not just an abstract opportunity but a clear reason to seek a first-instance decision on a request that it had not introduced until the appeal stage.

5.2.2 Request not submitted by proprietor in opposition proceedings

(CLB, IV.E.4.3.2 b))

In T 872/09 none of the patentee's auxiliary requests were admitted into the proceedings because the patentee, during the first-instance opposition proceedings, deliberately chose not to defend any single auxiliary request, even though it was aware of the fact that its main request had not been found allowable by the opposition division (points 2 and 3 of the Reasons).

In T 1400/11 the appellant (patent proprietor) did not react at all to the opposition filed against its patent, although all the reasons for revocation of the opposed patent were known. Thus, the appellant effectively circumvented the opposition proceedings by defending its patent exclusively in the appeal proceedings. In this way, the appellant had tried to shift its case entirely to the second instance, thereby compelling the board either to give a first ruling on the appellant's case or to remit the matter to the opposition division. Such a shifting of the appellant's case would also put the respondent at an unjustified disadvantage because if the appellant's main request were admitted and decided upon by the board, the respondent would be factually deprived of an examination of that issue before two instances. Since there was no admissible request of the appellant on which the appeal proceedings could be based, the appeal was dismissed.

Also in T 1616/10 the main request and auxiliary request 1 were not admitted into the proceedings, because claim 1 was broader than all requests underlying the appealed decision and the requests could have been filed in first-instance proceedings.

5.2.3 Resubmission of requests withdrawn at opposition stage

(CLB, IV.E.4.3.2 c))

In T 2599/11 the board admitted the sole request into the appeal proceedings that was withdrawn in opposition proceedings. The board was of the opinion that the purpose of the appeal proceedings as defined in G 9/91 and G 10/91 (OJ 1993, 408, 420) did not prevent the appellant from submissions and requests which, after revocation of its patent by the opposition division, reverted to a more broadly worded version in appeal proceedings than the claims defended in opposition proceedings, if these broader claims did not amount to a completely fresh case.

With respect to Art. 12(4) RPBA the board had to consider whether the non-filing of the sole request or the withdrawal of the similar request A in the opposition proceedings prevented the opposition division from giving a reasoned decision on the critical issues (cf. Case Law of the Boards of Appeal of the EPO, 7th ed. 2013, IV.E.4.3.2 c)).

In this context, the respondent, referring to decision T 361/08, had argued that the first auxiliary request filed with the statement of grounds of appeal was to be rejected as inadmissible. In that decision (point 13 of the Reasons) the board had stated that the fact alone that the main request in question was submitted and subsequently withdrawn in the first-instance proceedings was sufficient reason not to admit this request into appeal proceedings. The present board, however, did not subscribe to this general statement but it endorsed the more restricted statement, in the third paragraph of point 14 of the Reasons, that it is not the patentee's right to revert to granted claims if these claims did not form a basis for the decision under appeal because the request comprising these claims was withdrawn in the first-instance proceedings.

The board concluded that since all features of claim 1 of the sole request were present in claim 1 of request E which formed the basis of the impugned decision and on which the opposition division came to a negative finding on its subject-matter regarding inventive step, the opposition division had also expressed itself – at least implicitly – on the subject-matter present in the sole request. The board noted that the critical issues in the opposition and the appeal proceedings were identical in so far as the question had to be answered whether the claimed subject-matter involved an inventive step. Hence, the sole request of the appellant did not bring an entirely fresh case and the decision of the board could be taken on the basis of the subject of dispute in the first instance proceedings. For these reasons the sole request was admitted into the proceedings.

In T 1697/12 the board used its discretionary power according to Art. 12(4) RPBA not to admit the main request into the appeal proceedings which had been withdrawn during oral proceedings before the opposition division. The board stated that even though it might not have been the proprietor's intention to avoid a decision by the opposition division, the inevitable result of the withdrawal of the request was that a decision thereon was avoided since it was not thereby the subject of the reasoned decision of the opposition division. The patent proprietor had argued that in T 937/11 an auxiliary request, which had been withdrawn before the opposition division, was re-filed and admitted in appeal. The board held that the case underlying T 937/11 was different from the present one. The auxiliary request in that case contained the same critical issue as the main request before the opposition division; the withdrawal of the auxiliary request did not therefore have the effect of avoiding a decision of the opposition division on this issue, since it was decided when issuing a reasoned decision on the main request.

5.2.4 Amendments justified by the normal development of the proceedings

(CLB, IV.E.4.3.2 d))

In T 2485/11 the board confirmed the established jurisprudence that amendments, including amended requests, which can be considered as a normal reaction of a losing party given the circumstances, are usually allowed into the appeal proceedings by the boards (T 848/09, T 1072/98, T 540/01).

In the case at hand, the respondent (opponent) objected to the admission of auxiliary requests 1 to 5 on the grounds that these requests could already have been filed before the opposition division. In support of its arguments, the respondent cited decisions R 11/11 and T 936/09. These arguments did not convince the board for the following reasons:

Decision R 11/11 was based on decision T 144/09, in which the board did not admit sets of claims which could have been filed before the department of first instance pursuant to Art. 12(4) RBPA. At oral proceedings before the opposition division the patent proprietor was informed that the introduction of a particular feature contravened Art. 123(2) EPC. The patent proprietor did not file any requests in an attempt to overcome this objection despite an invitation to do so by the chairman of the opposition division. The board considered that claims with such a straightforward amendment could and should have been filed before the department of first instance, at least in the form of auxiliary requests, and therefore declined to admit them under Art. 12(4) RPBA.

In the case underlying decision T 936/09, the patent proprietor did not respond in substance to the opposition, despite having been given several opportunities to do so by the opposition division, but rather presented its case for the first time during the appeal proceedings.

By contrast, in the case at issue, the appellant had filed an amended main request and an auxiliary request during first-instance proceedings in an attempt to overcome objections of lack of novelty and inventive step raised in the notice of opposition. At the oral proceedings, the opposition division concluded that the amended requests did not comply with Art. 123(2) EPC. The appellant, unlike in T 144/09 and T 936/09, took the opportunity offered to it and filed further requests modifying the amendments in the main request, in order to overcome the opposition division's objection under Art. 123(2) EPC. The fact that, after these requests had also been considered to contravene Art. 123(2) EPC, the appellant did not file a further request could not be regarded as an abuse simply because, theoretically, it could have filed amendments. Moreover, the auxiliary requests were filed at the earliest possible stage in the appeal proceedings, namely with the statement of grounds of appeal. Hence, given the circumstances of the case, the appellant's submission of auxiliary requests 1 to 5 was considered to be a normal and legitimate reaction of a losing party (see also similar case T 2244/11).

5.2.5 Consideration of amended claims – number of auxiliary requests

(CLB, IV.E.4.4.9)

In T 280/11 the board found that the patent proprietor, which in written proceedings had filed a very high number of requests (a main request and 681 auxiliary requests, 454 of them just three days before the oral proceedings) and had submitted a further four auxiliary requests during the oral proceedings, had to expect a significant portion of the time available on the day of the proceedings to be taken up solely with the administrative handling of all the 686 requests. After the board had discussed and rejected these requests at the oral proceedings, or (in the case of new auxiliary requests 682 to 685) after lengthy debate had refused to admit them into the proceedings, the proprietor could not reasonably expect to be granted a further break late in the afternoon of the same day to allow further requests to be drafted and submitted (point 5 of the Reasons).

6. Reimbursement of appeal fees

6.1 Referral G 1/14

(CLB, IV.E.8.1.2)

In T 1553/13 (JO 2014, A84) the appeal was filed after the time limit under Art. 108, first sentence, EPC had expired, and the appeal fee was also paid late. Under Art. 108, second sentence, EPC notice of appeal is not deemed to have been filed until the fee for appeal has been paid. There have been numerous board decisions ruling that there is no appeal unless the appeal fee is paid within the time limit. From this it is inferred that an appeal fee paid after the time limit expires is to be reimbursed. In landmark decision J 16/82 (OJ 1983, 262) the appeal was deemed not to have been filed owing to late payment of the appeal fee, and the fee was reimbursed. Similarly, in J 21/80 (OJ 1981, 101) the appeal fee was reimbursed on the grounds that in the circumstances the purpose of the fee payment could no longer be achieved. On the other hand in T 79/01, T 1289/10, T 1535/10 and T 2210/10, where the appeal fee had also been paid late, the appeal was dismissed as inadmissible but reimbursement of the appeal fee was not ordered.

The wording that appears in Art. 108, second sentence, EPC is also used in many other provisions, such as R. 136(1), third sentence, EPC (request for re-establishment). Hence the interpretation of Art. 108, second sentence, EPC might have repercussions reaching beyond the case in point.

For these reasons the following point of law was referred to the Enlarged Board of Appeal (referral G 1/14):

If, after expiry of the time limit under Art. 108, first sentence, EPC a notice of appeal is filed and the fee for appeal is paid, is the appeal inadmissible or is it deemed not to have been filed?

6.2 Reimbursement must be equitable

(CLB, IV.E.8.5.1)

In T 2106/10, the board acknowledged that several deficiencies occurred in the first-instance proceedings, but reiterated that any reimbursement of the appeal fee had to be equitable and depended on the question whether these procedural deficiencies were 1) substantial and 2) forced the appellant to file the present appeal. None of the alleged procedural deficiencies raised by the appellant were specifically related to the refusal of the main request. As the appellant pursued the granting of a patent on the basis of the main request, he had to file the appeal irrespective of whether or not the alleged substantial procedural violations had occurred. Therefore, the board rejected the appellant's request for reimbursement and could leave open whether or not a substantial procedural violation had occurred.

7. Proceedings before the Enlarged Board of Appeal

7.1 Referral under Article 112 EPC – admissibility

(CLB, IV.E.9.1)

G 1/12 (OJ 2014, A114; see also Chapter IV.E.1.3.1 a) "Rule 99(1)(a) EPC" and Chapter III.E.2 "Evaluation of evidence") concerns the correction of errors regarding the identity of the appellant. The Enlarged Board held that the question concerning the admissibility of an appeal (filed by a person appearing at first sight not to have standing to do so) relates to a "point of law of fundamental importance" because it is relevant in a potentially large number of cases and therefore of great interest not only to the parties to the specific appeal proceedings in question. Moreover, settling this point of law is important not only to the users of the European patent system but also to the boards of appeal and the department of first instance in examination and opposition proceedings. The Enlarged Board also considered the requirement of "uniform application of the law" to be fulfilled. Some board decisions allowed deficiencies in the appellant's name to be remedied under R. 101(2) EPC, whereas in other decisions on analogous situations the boards applied R. 139 EPC. The Enlarged Board concluded that the referral was admissible.

A minority of the members of the Enlarged Board of Appeal considered the referral inadmissible. The minority did not share the majority's view that "a point of law is also to be regarded as fundamental, if its importance extends beyond the specific case in hand" and that such importance is established "if the point of law could be relevant to a large number of similar cases". This view implied that "importance" within the meaning of Art. 112 EPC would be nothing more than mere relevance. The number of cases affected, however, would be neither a suitable nor an appropriate criterion for establishing the admissibility of a referral. The minority was also of the opinion that the law had been clear at least since decision T 97/98 (OJ 2002, 183). The fact that some board decisions allowed deficiencies in the appellant's name to be remedied under R. 101(2) EPC whereas in other decisions the boards applied R. 139 EPC merely demonstrated that, as long as the removal of the deficiency does not lead to a change of the (original) appellant's true identity, both procedures are available according to the consistent case law. Although the minority agreed with the answers to the referred questions, it concluded that the requirements of Art. 112(1)(a) EPC were not fulfilled.

In G 1/13 (OJ 2015, A42; see also chapter IV.D.1 "Admissibility of opposition" and Chapter IV.E.1.3.1 "Form and content of notice of appeal (R. 99(1) EPC)"), the Enlarged Board of Appeal confirmed that a point of law is to be regarded as of fundamental importance if its impact extends beyond the specific case at hand and, for example, if it could be relevant to a large number of similar cases and/or is important not only to the users of the European patent system but also to the boards of appeal and the EPO itself.

7.2 Petition for review under Article 112a EPC

7.2.1 Objection not complying with Rule 106 EPC

(CLB, IV.E.9.2.5 c))

In R 18/12, the board held that only an objection which, in substance, raises a procedural defect which can be the subject of a petition for review under Art. 112a(2)(a) to (d) EPC can be regarded as an objection within the meaning of R. 106 EPC. R. 106 EPC is not a formality. Its essential purpose is to avoid unnecessary petitions for review by giving the board the possibility to correct a defect which could otherwise lead to review proceedings. An objection must be drafted in such a way that the board is able to determine whether it is directed at a procedural defect or at an allegedly wrong assessment of substantive issues, which falls outside the scope of review proceedings. The crucial factor for the assessment of whether the petitioner has observed R. 106 EPC is the objection's substance, not its formal wording. In the case at hand, although mentioning Art. 113 EPC, the petitioner only objected to the board's conclusion on lack of prima facie clarity. Therefore, it did not raise an objection within the meaning of R. 106 EPC. The petition was rejected as clearly inadmissible.

7.2.2 Fundamental violation of Article 113 EPC

(CLB, IV.E.9.2.9)

In R 16/12, the petitioner put forward that the board had only discussed sufficiency of disclosure during the oral proceedings. The petitioner claimed that the arguments discussed under Art. 83 and 100(b) EPC related rather to the assessment of inventive step, and that he had been denied the opportunity to present his arguments under Art. 56 and 100(a) EPC. The petitioner essentially argued that if a patent proprietor requests to discuss a particular ground for opposition, it should be allowed to do so. The Enlarged Board disagreed, stating that such an entitlement is not supported by the principle of party disposition, which relates merely to the parties' right to dispose of their requests by advancing, withholding or withdrawing them as they see fit. Moreover, the principle of efficiency required a board of appeal to focus on those points that are relevant for the decision. Since the board considered that the case could be decided in respect of the issue of sufficiency of disclosure as ratio decidendi, discussing any other ground would have concerned an issue that did not form a necessary part of the board's final decision (obiter dictum). The petition was rejected as clearly unallowable.

In R 3/13, the Enlarged Board agreed that the right to be heard may be infringed if a decision is based on grounds or evidence which surprise the adversely affected party because they were not discussed in the proceedings. This is clear from the wording of Art. 113(1) EPC which requires that the decisions of the EPO may only be based on grounds or evidence on which the parties concerned have had an opportunity to present their comments. This implies that a party may not be taken by surprise by the reasons for a decision, referring to unknown grounds or evidence. In the case at issue, however, the issue of an intermediate generalisation incompatible with Art. 123(2) EPC had already featured in the earlier opposition proceedings, as well as in the board's communication annexed to the summons to oral proceedings. The petitioner's subjective feeling of surprise did not alter the fact that it was aware of the objections raised against its requests and had also been able to comment on them. The Enlarged Board also stressed that the case law consistently holds that a board would fail in its obligations of impartiality if one party were to be repeatedly advised that its requests were not patentable, until it managed to come up with a grantable claim. For these reasons, the petition was dismissed as clearly unallowable.

In R 5/13 (see also consolidated proceedings R 9/13, R 10/13, R 11/13, R 12/13 and R 13/13; see also Chapter I.C.2 "Closest prior art") the petitioners' key objection was that the board had limited the discussion of inventive step to document D2 as the (only) starting point when applying the problem solution approach. When the board refused a discussion of inventive step starting from document D1, it deprived the petitioners of the opportunity to present their arguments. The Enlarged Board considered the board's reasoning that "taking document D1 as a starting point for the analysis of inventive step relies on a hindsight knowledge of what is claimed" to be a matter of substantive law. Petition for review proceedings, however, cannot be used to review the exercise of a board's discretionary power if that would involve an impermissible consideration of substantive issues. The Enlarged Board rejected the petition as clearly unallowable.

V. PROCEEDINGS BEFORE THE DISCIPLINARY BOARD OF APPEAL

1. Appeals against decisions of the Examination Board

1.1 Objective review of the marks awarded

(CLB, V.2.6.3)

In case D 3/14 the appellant had obtained the grade "fail" (with 68 marks) in the pre-examination for the European qualifying examination. She argued that the answer to statement 10.4 in the question paper should have been "false", rather than "true", as given in the Examiner's Report. She requested that the decision of the Examination Board be set aside and that she be awarded a "pass".

Request that the decision be set aside

The board recalled that decisions of the Examination Board may in principle only be reviewed to establish that they do not infringe the REE, the provisions relating to its application, or higher-ranking law (Art. 24(1) REE; consistent case law following D 1/92). It was not the function of the board to reconsider the entire examination procedure on the merits, since the Examination Committee and the Examination Board had some latitude of evaluation, subject to only limited judicial review. Only if the appellant could show that the contested decision was based on serious and obvious mistakes could the board take this into account. The alleged mistake must be so obvious that it could be established without reopening the entire marking procedure.

According to the board, statement 10.4 should have been formulated differently in order to express the aim intended. Candidates could not be expected to make an assumption in this respect (cf. R. 22(3) IPREE, Supplementary publication 2, OJ EPO 2014, p. 18, providing that candidates must limit themselves to the facts given in the examination paper). On the board's analysis the examiner's evaluation of the examination papers rested on a question that could not, on an objective reading, be derived from statement 10.4. The appellant's understanding of statement 10.4 was justified from an objective point of view. The answer to it was "false" and not "true" as indicated in the examiner's report, and the discrepancy between the underlying question and the expected answer was to the disadvantage of the appellant. The examiners had based their evaluation on an incorrect premise and therefore the contested decision was based on serious and obvious mistakes which could be established without reopening the entire marking procedure. The decision had to be set aside and the appeal fee reimbursed (Art. 24(4) REE).

The board also took issue with the Examination Board's failure to rectify its decision (Art. 24(3) REE). The Examination Board was obliged to assess carefully whether or not the relevant requirements were met before deciding to grant or refuse rectification (D 38/05, D 4/06). In clear cases rectification was a quick and simple way of cancelling flawed decisions, which spared parties the cost, in time and money, of appeal proceedings and was thus in the public interest and the interest of the appellant (D 38/05, D 4/06). Having regard to the obvious discrepancy in the present case, rectification had been warranted.

Request that the decision be corrected

The board pointed out that the it was not empowered by Art. 24(4), second sentence, REE to correct the decision, i.e. to review the marks and grade of an examination paper. It therefore considered whether special reasons provided a legal basis for not remitting the case to the Examination Board (cf. Art. 12 Additional Rules of Procedure of the DBA, Supplementary publication 1, OJ EPO 2014, p. 54). Only in a few exceptional cases had the DBA found such special reasons to exist (D 5/86, D 11/91, D 8/08 and D 9/08), but in appeals against decisions of the Examination Board it had not made use of this power to review the contested decision on its merits (on the limits of judicial review by the board see D 7/05).

The present situation differed from circumstances relating to complaints about the marking of examination papers which underlay the established jurisprudence. The pre-examination was a multiple-choice paper. The examiners had no latitude when assessing the correctness of the answers; the marks were awarded according to a strict scheme not involving any discretion and the award of a "pass" or "fail" grade was merely the arithmetical outcome of the marks achieved. From a limited review of the decision case, the board was able to establish the correct marks on the basis of the appellant's answer paper without interfering with any value judgement of the Examination Committee or Examination Board. Also taken into account were the matter's urgency, since a "pass" in the pre-examination was a precondition for the main examination, and the fact that the Examining Board, by not rectifying its decision, even though the discrepancy had been comprehensively substantiated, had burdened the appellant with appeal proceedings.

These were special reasons which justified the board scrutinising the appellant's marks and deciding whether the grade was to be revised. Taking into account the correction with respect to statement 10.4, the total marks rose from 68 to 70 and the "pass" grade was therefore awarded.

VI. INSTITUTIONAL MATTERS

1. Amendments to the Rules of Procedure of the Boards of Appeal – legislative power with the Administrative Council

(CLB, VII.2)

In T 1100/10, in the light of the board's decision not to admit a number of requests into the proceedings, appellant I raised a point of law with regard to the legal basis and the legality of the Rules of Procedure of the Boards of Appeal (RPBA) and requested the referral of four questions to the Enlarged Board of Appeal (EBA) under Art. 112 EPC. The appellant argued that the RPBA had no legal basis because they were drafted by the Presidium of the boards of appeal, which had no legislative power to do so. The Presidium was not a department mentioned in Art. 15 EPC, and drafting the RPBA went far beyond the role and function of the Presidium as established in R. 12(3) EPC. According to R. 12(3) EPC, the Presidium was only entitled to adopt the RPBA. This was entirely different from drafting and issuing the RPBA. The function of the Presidium under R. 12(3) EPC did not include matters such as the drafting of legal norms and/or rules that could affect parties in appeal proceedings.

The board was of the opinion that under Art. 23(4) EPC it is clearly within the power of the Presidium to draft the RPBA and to adopt a version of these rules to be proposed for the approval of the Administrative Council. It is the Administrative Council of the European Patent Organisation which decides to approve the RPBA, which only thereby come into force and thus can subsequently be applied by the boards of appeal. Any further amendments to the RPBA must also be approved by the Administrative Council. Thus, legislative power clearly lies with the Administrative Council. The board noted that Art. 112(1)(a) EPC concerns, inter alia, the possibility of referring questions to the EBA in cases where there is an important point of law. The board pointed out that questions based on unsubstantiated assumptions and hypothetical considerations are not suitable for a referral to the EBA (cf. T 1213/05 and T 118/89). Therefore, the appellant's request for referral to the EBA was refused.

 

178 references found.

Click X to load a reference inside the current page, click on the title to open in a new page.

PCT Articles

Offical Journal of the EPO

Case Law of the Enlarged Board

General Case Law